Swimming in space. What is the mechanism?










17












$begingroup$


A large (compared to Earth life) species of creatures live in space. They gain energy from interstellar dust and starlight. How they do this is a separate question so not relevant here. However they can adjust their metabolism to eat enormous amounts quickly when food is available or hibernate for thousands of years if it isn't.



The question is, how do they move around.



So far I have:



  1. Solar sails. They spread their 'wings' and are pushed by radiation from nearby stars. They live virtually for ever, so time and speed aren't a problem.


  2. They can store faeces indefinitely and when they expel them they do so in a controlled way that allows acceleration and braking and course changes.


  3. They also 'swim' or 'fly' through space by flapping or moving their extremities.


Question



According to Newton the third option (swimming) is impossible in a vacuum if it is done simply by using bodily movements.



So what discoveries in physics since Newton will enable them to move this way?



I'm thinking of the ideas that even a vacuum is not completely empty and that dark matter and energy exist. Also - Is there enough ordinary 'stuff'
such as interstellar gas for them to swim through, perhaps even gaining enough momentum to travel from one gas cloud to another, drifting between them?



EDITS



  1. The creatures are completely flexible in the manner of an amoeba. They can assume almost any shape and can expand to a flat disk a few kilometres in diameter or contract to a ball shape.


  2. I think I haven't made it entirely clear. While the creatures may have other methods of propulsion (and in fact they do as mentioned above), I particularly want them to swim through space as well. They can be seen majestically pulsing along looking a lot like some vast version of a sea-creature on Earth.










share|improve this question











$endgroup$







  • 1




    $begingroup$
    Only magic or fictional science method would enable them to swim. But they can have something like an Ion engine. Why they look like they swim? Coz they like it! What else you would do defore next snack.
    $endgroup$
    – Artemijs Danilovs
    Nov 14 '18 at 1:09






  • 3




    $begingroup$
    You can actually "swim" in space, but only in certain regions, link
    $endgroup$
    – SilverCookies
    Nov 14 '18 at 8:58










  • $begingroup$
    SilverCookies - This is what I'm looking for. The creatures can get from one such region to another by accelerating in the right direction and then going into hibernation. Could you work this into an answer? Don't worry, I don't expect you to buy the scientific paper. Do you know what or where these regions are and how far apart? If the regions are big enough, maybe the creatures could spend a whole lifetime in such a region.
    $endgroup$
    – chasly from UK
    Nov 14 '18 at 10:26







  • 1




    $begingroup$
    @chaslyfromUK I can work an answer after work if you want (I actually have the paper) but the resulting creature can only swim in unusual places like around a black-hole/worm-hole or perhaps a neutron star; else it would need to be huge...
    $endgroup$
    – SilverCookies
    Nov 14 '18 at 13:12







  • 1




    $begingroup$
    That sounds excellent. I don't mind where the species is restricted to. Maybe it's only native to one particular star. Possibly it reproduces by exploding into numerous spores that stay nearby or even travel through the vastness of space. I don't even mind what size the creatures are as long as they are visible enough for FTL tourists to come on 'whale-watching' tours. In order to see the 'whales' I guess the star needs to emit visible light and the creatures themselves will need to receive radiated energy.
    $endgroup$
    – chasly from UK
    Nov 14 '18 at 13:30















17












$begingroup$


A large (compared to Earth life) species of creatures live in space. They gain energy from interstellar dust and starlight. How they do this is a separate question so not relevant here. However they can adjust their metabolism to eat enormous amounts quickly when food is available or hibernate for thousands of years if it isn't.



The question is, how do they move around.



So far I have:



  1. Solar sails. They spread their 'wings' and are pushed by radiation from nearby stars. They live virtually for ever, so time and speed aren't a problem.


  2. They can store faeces indefinitely and when they expel them they do so in a controlled way that allows acceleration and braking and course changes.


  3. They also 'swim' or 'fly' through space by flapping or moving their extremities.


Question



According to Newton the third option (swimming) is impossible in a vacuum if it is done simply by using bodily movements.



So what discoveries in physics since Newton will enable them to move this way?



I'm thinking of the ideas that even a vacuum is not completely empty and that dark matter and energy exist. Also - Is there enough ordinary 'stuff'
such as interstellar gas for them to swim through, perhaps even gaining enough momentum to travel from one gas cloud to another, drifting between them?



EDITS



  1. The creatures are completely flexible in the manner of an amoeba. They can assume almost any shape and can expand to a flat disk a few kilometres in diameter or contract to a ball shape.


  2. I think I haven't made it entirely clear. While the creatures may have other methods of propulsion (and in fact they do as mentioned above), I particularly want them to swim through space as well. They can be seen majestically pulsing along looking a lot like some vast version of a sea-creature on Earth.










share|improve this question











$endgroup$







  • 1




    $begingroup$
    Only magic or fictional science method would enable them to swim. But they can have something like an Ion engine. Why they look like they swim? Coz they like it! What else you would do defore next snack.
    $endgroup$
    – Artemijs Danilovs
    Nov 14 '18 at 1:09






  • 3




    $begingroup$
    You can actually "swim" in space, but only in certain regions, link
    $endgroup$
    – SilverCookies
    Nov 14 '18 at 8:58










  • $begingroup$
    SilverCookies - This is what I'm looking for. The creatures can get from one such region to another by accelerating in the right direction and then going into hibernation. Could you work this into an answer? Don't worry, I don't expect you to buy the scientific paper. Do you know what or where these regions are and how far apart? If the regions are big enough, maybe the creatures could spend a whole lifetime in such a region.
    $endgroup$
    – chasly from UK
    Nov 14 '18 at 10:26







  • 1




    $begingroup$
    @chaslyfromUK I can work an answer after work if you want (I actually have the paper) but the resulting creature can only swim in unusual places like around a black-hole/worm-hole or perhaps a neutron star; else it would need to be huge...
    $endgroup$
    – SilverCookies
    Nov 14 '18 at 13:12







  • 1




    $begingroup$
    That sounds excellent. I don't mind where the species is restricted to. Maybe it's only native to one particular star. Possibly it reproduces by exploding into numerous spores that stay nearby or even travel through the vastness of space. I don't even mind what size the creatures are as long as they are visible enough for FTL tourists to come on 'whale-watching' tours. In order to see the 'whales' I guess the star needs to emit visible light and the creatures themselves will need to receive radiated energy.
    $endgroup$
    – chasly from UK
    Nov 14 '18 at 13:30













17












17








17


2



$begingroup$


A large (compared to Earth life) species of creatures live in space. They gain energy from interstellar dust and starlight. How they do this is a separate question so not relevant here. However they can adjust their metabolism to eat enormous amounts quickly when food is available or hibernate for thousands of years if it isn't.



The question is, how do they move around.



So far I have:



  1. Solar sails. They spread their 'wings' and are pushed by radiation from nearby stars. They live virtually for ever, so time and speed aren't a problem.


  2. They can store faeces indefinitely and when they expel them they do so in a controlled way that allows acceleration and braking and course changes.


  3. They also 'swim' or 'fly' through space by flapping or moving their extremities.


Question



According to Newton the third option (swimming) is impossible in a vacuum if it is done simply by using bodily movements.



So what discoveries in physics since Newton will enable them to move this way?



I'm thinking of the ideas that even a vacuum is not completely empty and that dark matter and energy exist. Also - Is there enough ordinary 'stuff'
such as interstellar gas for them to swim through, perhaps even gaining enough momentum to travel from one gas cloud to another, drifting between them?



EDITS



  1. The creatures are completely flexible in the manner of an amoeba. They can assume almost any shape and can expand to a flat disk a few kilometres in diameter or contract to a ball shape.


  2. I think I haven't made it entirely clear. While the creatures may have other methods of propulsion (and in fact they do as mentioned above), I particularly want them to swim through space as well. They can be seen majestically pulsing along looking a lot like some vast version of a sea-creature on Earth.










share|improve this question











$endgroup$




A large (compared to Earth life) species of creatures live in space. They gain energy from interstellar dust and starlight. How they do this is a separate question so not relevant here. However they can adjust their metabolism to eat enormous amounts quickly when food is available or hibernate for thousands of years if it isn't.



The question is, how do they move around.



So far I have:



  1. Solar sails. They spread their 'wings' and are pushed by radiation from nearby stars. They live virtually for ever, so time and speed aren't a problem.


  2. They can store faeces indefinitely and when they expel them they do so in a controlled way that allows acceleration and braking and course changes.


  3. They also 'swim' or 'fly' through space by flapping or moving their extremities.


Question



According to Newton the third option (swimming) is impossible in a vacuum if it is done simply by using bodily movements.



So what discoveries in physics since Newton will enable them to move this way?



I'm thinking of the ideas that even a vacuum is not completely empty and that dark matter and energy exist. Also - Is there enough ordinary 'stuff'
such as interstellar gas for them to swim through, perhaps even gaining enough momentum to travel from one gas cloud to another, drifting between them?



EDITS



  1. The creatures are completely flexible in the manner of an amoeba. They can assume almost any shape and can expand to a flat disk a few kilometres in diameter or contract to a ball shape.


  2. I think I haven't made it entirely clear. While the creatures may have other methods of propulsion (and in fact they do as mentioned above), I particularly want them to swim through space as well. They can be seen majestically pulsing along looking a lot like some vast version of a sea-creature on Earth.







xenobiology space-travel science






share|improve this question















share|improve this question













share|improve this question




share|improve this question








edited Nov 14 '18 at 0:24







chasly from UK

















asked Nov 13 '18 at 18:11









chasly from UKchasly from UK

14.1k465133




14.1k465133







  • 1




    $begingroup$
    Only magic or fictional science method would enable them to swim. But they can have something like an Ion engine. Why they look like they swim? Coz they like it! What else you would do defore next snack.
    $endgroup$
    – Artemijs Danilovs
    Nov 14 '18 at 1:09






  • 3




    $begingroup$
    You can actually "swim" in space, but only in certain regions, link
    $endgroup$
    – SilverCookies
    Nov 14 '18 at 8:58










  • $begingroup$
    SilverCookies - This is what I'm looking for. The creatures can get from one such region to another by accelerating in the right direction and then going into hibernation. Could you work this into an answer? Don't worry, I don't expect you to buy the scientific paper. Do you know what or where these regions are and how far apart? If the regions are big enough, maybe the creatures could spend a whole lifetime in such a region.
    $endgroup$
    – chasly from UK
    Nov 14 '18 at 10:26







  • 1




    $begingroup$
    @chaslyfromUK I can work an answer after work if you want (I actually have the paper) but the resulting creature can only swim in unusual places like around a black-hole/worm-hole or perhaps a neutron star; else it would need to be huge...
    $endgroup$
    – SilverCookies
    Nov 14 '18 at 13:12







  • 1




    $begingroup$
    That sounds excellent. I don't mind where the species is restricted to. Maybe it's only native to one particular star. Possibly it reproduces by exploding into numerous spores that stay nearby or even travel through the vastness of space. I don't even mind what size the creatures are as long as they are visible enough for FTL tourists to come on 'whale-watching' tours. In order to see the 'whales' I guess the star needs to emit visible light and the creatures themselves will need to receive radiated energy.
    $endgroup$
    – chasly from UK
    Nov 14 '18 at 13:30












  • 1




    $begingroup$
    Only magic or fictional science method would enable them to swim. But they can have something like an Ion engine. Why they look like they swim? Coz they like it! What else you would do defore next snack.
    $endgroup$
    – Artemijs Danilovs
    Nov 14 '18 at 1:09






  • 3




    $begingroup$
    You can actually "swim" in space, but only in certain regions, link
    $endgroup$
    – SilverCookies
    Nov 14 '18 at 8:58










  • $begingroup$
    SilverCookies - This is what I'm looking for. The creatures can get from one such region to another by accelerating in the right direction and then going into hibernation. Could you work this into an answer? Don't worry, I don't expect you to buy the scientific paper. Do you know what or where these regions are and how far apart? If the regions are big enough, maybe the creatures could spend a whole lifetime in such a region.
    $endgroup$
    – chasly from UK
    Nov 14 '18 at 10:26







  • 1




    $begingroup$
    @chaslyfromUK I can work an answer after work if you want (I actually have the paper) but the resulting creature can only swim in unusual places like around a black-hole/worm-hole or perhaps a neutron star; else it would need to be huge...
    $endgroup$
    – SilverCookies
    Nov 14 '18 at 13:12







  • 1




    $begingroup$
    That sounds excellent. I don't mind where the species is restricted to. Maybe it's only native to one particular star. Possibly it reproduces by exploding into numerous spores that stay nearby or even travel through the vastness of space. I don't even mind what size the creatures are as long as they are visible enough for FTL tourists to come on 'whale-watching' tours. In order to see the 'whales' I guess the star needs to emit visible light and the creatures themselves will need to receive radiated energy.
    $endgroup$
    – chasly from UK
    Nov 14 '18 at 13:30







1




1




$begingroup$
Only magic or fictional science method would enable them to swim. But they can have something like an Ion engine. Why they look like they swim? Coz they like it! What else you would do defore next snack.
$endgroup$
– Artemijs Danilovs
Nov 14 '18 at 1:09




$begingroup$
Only magic or fictional science method would enable them to swim. But they can have something like an Ion engine. Why they look like they swim? Coz they like it! What else you would do defore next snack.
$endgroup$
– Artemijs Danilovs
Nov 14 '18 at 1:09




3




3




$begingroup$
You can actually "swim" in space, but only in certain regions, link
$endgroup$
– SilverCookies
Nov 14 '18 at 8:58




$begingroup$
You can actually "swim" in space, but only in certain regions, link
$endgroup$
– SilverCookies
Nov 14 '18 at 8:58












$begingroup$
SilverCookies - This is what I'm looking for. The creatures can get from one such region to another by accelerating in the right direction and then going into hibernation. Could you work this into an answer? Don't worry, I don't expect you to buy the scientific paper. Do you know what or where these regions are and how far apart? If the regions are big enough, maybe the creatures could spend a whole lifetime in such a region.
$endgroup$
– chasly from UK
Nov 14 '18 at 10:26





$begingroup$
SilverCookies - This is what I'm looking for. The creatures can get from one such region to another by accelerating in the right direction and then going into hibernation. Could you work this into an answer? Don't worry, I don't expect you to buy the scientific paper. Do you know what or where these regions are and how far apart? If the regions are big enough, maybe the creatures could spend a whole lifetime in such a region.
$endgroup$
– chasly from UK
Nov 14 '18 at 10:26





1




1




$begingroup$
@chaslyfromUK I can work an answer after work if you want (I actually have the paper) but the resulting creature can only swim in unusual places like around a black-hole/worm-hole or perhaps a neutron star; else it would need to be huge...
$endgroup$
– SilverCookies
Nov 14 '18 at 13:12





$begingroup$
@chaslyfromUK I can work an answer after work if you want (I actually have the paper) but the resulting creature can only swim in unusual places like around a black-hole/worm-hole or perhaps a neutron star; else it would need to be huge...
$endgroup$
– SilverCookies
Nov 14 '18 at 13:12





1




1




$begingroup$
That sounds excellent. I don't mind where the species is restricted to. Maybe it's only native to one particular star. Possibly it reproduces by exploding into numerous spores that stay nearby or even travel through the vastness of space. I don't even mind what size the creatures are as long as they are visible enough for FTL tourists to come on 'whale-watching' tours. In order to see the 'whales' I guess the star needs to emit visible light and the creatures themselves will need to receive radiated energy.
$endgroup$
– chasly from UK
Nov 14 '18 at 13:30




$begingroup$
That sounds excellent. I don't mind where the species is restricted to. Maybe it's only native to one particular star. Possibly it reproduces by exploding into numerous spores that stay nearby or even travel through the vastness of space. I don't even mind what size the creatures are as long as they are visible enough for FTL tourists to come on 'whale-watching' tours. In order to see the 'whales' I guess the star needs to emit visible light and the creatures themselves will need to receive radiated energy.
$endgroup$
– chasly from UK
Nov 14 '18 at 13:30










17 Answers
17






active

oldest

votes


















24












$begingroup$

Space is not perfectly empty. There are some sparse atoms and particles, which, under the right conditions, can be used to exert a force and use Newton 3rd law.



If you want to use fluid dynamic forces in space, you just need big wings. How big?



This answer gives an estimate:




Because of the low density of particles in the interstellar medium, a normal-sized wing will not act as an airfoil. It will be more like bouncing tennis balls off the wing every so often.



The Knudsen number quantifies when fluid dynamics takes over: Kn = mean_free_path / airfoil_length_scale. Knudsen numbers greater than about 10 are in the regime of ballistic collisions, rather than fluid flow. The mean free path in the interstellar medium is huge (about 70 astronomical units). So to get fluid dynamics, you'd need an airfoil of size more than the Sun–Jupiter distance.







share|improve this answer









$endgroup$








  • 1




    $begingroup$
    I've added something about the size of the creatures. Is an air-foil actually necessary? That's why I called it swimming rather than flying. For example I don't believe an octopus or a jellyfish use air-foils to move through water. (or do they?)
    $endgroup$
    – chasly from UK
    Nov 13 '18 at 19:46











  • $begingroup$
    @chaslyfromUK, an octopus uses reaction (which you covered)
    $endgroup$
    – L.Dutch
    Nov 13 '18 at 19:50






  • 2




    $begingroup$
    Reaction against faeces yes. What about gulping interstellar gas and passing it through the body without absorption? Will that work?
    $endgroup$
    – chasly from UK
    Nov 13 '18 at 19:59






  • 2




    $begingroup$
    Ballistic is fine for pushing on matter with large flat panels, if you can move them as fast as you want to travel (relative to the medium). You can rotate them as you pull them forward to reduce their surface area to near-0, and to max for the downstroke. Or a spinning fan could maybe push on matter. Neither are very practical for a large living creature, and the low density just makes it all around impractical, but being in the ballistic regime is not a show stopper on its own. You could "swim" through a field of floating heavy tennis balls by on average pushing them backwards.
    $endgroup$
    – Peter Cordes
    Nov 14 '18 at 0:47






  • 1




    $begingroup$
    @PeterCordes: a matrix of wings might make more sense. I can imagine huge feathery fractaline cilia with billions of ‘rapidly’ moving wings all arrayed in a huge panel to provide thrust.
    $endgroup$
    – Joe Bloggs
    Nov 14 '18 at 13:55


















12












$begingroup$

Negative mass.



Each of your creatures contains a precious nugget of negative mass. When one of these creatures reproduces, the offspring are each given a fragment of the nugget. As they travel the universe, each creature accumulates any additional negative mass encountered and adds it to the nugget. When a creature dies, the nugget is claimed by its kin.



Negative mass can be used to propel. The larger and more unmassive this chunk is, the more useful it is for propulsion.



https://en.wikipedia.org/wiki/Negative_mass




For two positive masses, nothing changes and there is a gravitational
pull on each other causing an attraction. Two negative masses would
repel because of their negative inertial masses. For different signs
however, there is a push that repels the positive mass from the
negative mass, and a pull that attracts the negative mass towards the
positive one at the same time.



Hence Bondi pointed out that two objects of equal and opposite mass
would produce a constant acceleration of the system towards the
positive-mass object,[4] an effect called "runaway motion"...



Such a couple of objects would accelerate without limit (except
relativistic one); however, the total mass, momentum and energy of the
system would remain 0.




The nugget would be kept in the center of the body. When needing propulsion, the nugget would be moved to the end of an appendage. There it would repel the main body of the creature and be attracted at the same time. The same principle could be used to brake or steer by moving the nugget to a different position.






share|improve this answer









$endgroup$












  • $begingroup$
    I remember reading about negative mass in an old edition of New Scientist. I like this idea. The "swimming" motions could very well be how the creature resolves steering and acceleration changes.
    $endgroup$
    – Baldrickk
    Nov 14 '18 at 11:05










  • $begingroup$
    This sounds like fiction; am I correct in assuming it is? That's fine if so, obviously, but I just wanted to make sure. Simply stating "the total mass, momentum and energy of the system would remain 0" seems to assume only its own reference frame, as using 2 of these things would allow for perpetual motion machines with a net gain in energy.
    $endgroup$
    – Aaron
    Nov 14 '18 at 17:04






  • 2




    $begingroup$
    The maths of negative math is a known and real thing. It's never been shown to exist and as you point out has some.....problems... but so far as I know it hasn't been proven impossible either.
    $endgroup$
    – Tim B
    Nov 14 '18 at 17:31










  • $begingroup$
    @Aaron - follow the link and then keep going, if this interests you. The infamous Alcubierre drive relies on negative matter to work.
    $endgroup$
    – Willk
    Nov 14 '18 at 17:52






  • 1




    $begingroup$
    Unless the negative mass is equal to the positive mass, it doesn't matter where the mass is located in the body. The total net effect is to lower the overall mass.This makes other means of propulsion more effective, but gives no net propulsion itself. The force it generates is cancelled by the force used to hold it. The acceleration mentioned in the article only occurs when the negative and positive mass are exactly equal in magnitude. Otherwise, negative mass can only be used for acceleration by expelling it and allowing it to move off in the opposite direction.
    $endgroup$
    – Paul Sinclair
    Nov 14 '18 at 18:10



















11












$begingroup$

The creature is actually a giant bussard ramjet. It just looks like it's swimming, because the flaps are used to move particles from the surrounding area towards the body, where they are collected by polyp-like areas covering the body. They are then moved towards an organ that holds the particles until they are required for energy generation or used as propulsion matter. The body could also emit an EM field that moves particles closer to the body, so that the flaps can collect more of it.



The end result would be a movement similar to the breaststroke of a swimmer with a forward movement of the flaps close to the body to avoid pushing particles away and a wide backwards movement to collect a large volume of particles.



It's probably not scientifically viable, because the movement would most likely expand more energy than it could generate from what it collects, but it's at least science based. You could make the flaps solar energy collectors and have it hibernate during times where it's too far away from a sun to collect energy, if you want it's energy consumption to be slightly more realistic.






share|improve this answer











$endgroup$












  • $begingroup$
    L.Dutch covered the size of said ramjet in their answer. Not exactly, but the sale math applies I think. IANARS.
    $endgroup$
    – Draco18s
    Nov 14 '18 at 16:51






  • 1




    $begingroup$
    @Draco18s Nah, that's talking about different things. Bussard Ramjets use completely different principles and function at much more reasonable scales (although last I heard people thought they would probably work better as brakes than engines!)
    $endgroup$
    – Tim B
    Nov 14 '18 at 17:22






  • 1




    $begingroup$
    @TimB I haven't seen the science, and IANARS, so I'm just guessing. Point is, space is still ridiculously not-dense (the densest nebulae are less dense than the best laboratory vacuum chamber).
    $endgroup$
    – Draco18s
    Nov 14 '18 at 17:28






  • 1




    $begingroup$
    @Draco18s Wings use fluid dynamics, that's what the other answer covers. Bussard Ramjets use electromagnetic fields to gather interstellar medium to then use as fuel and propellant. It's a completely different mechanism.
    $endgroup$
    – Tim B
    Nov 14 '18 at 17:30


















5












$begingroup$

Surprisingly a creature swimming in space would be possible, in the sense that it would be able to change its position/location, but not is momentum which would make it useless for long distance travel.



The concept is called swimming in spacetime and is the only reactionless propulsion idea we have that would not violate physics.



According to wikipedia:




“Swimming in spacetime” is a geometrical motive principle that exploits the curved spacetime metric of the gravitational field to permit an extended body undergoing specific deformations in shape, to change position. In weak gravitational fields, like that of Earth, the change in position per deformation cycle would be far too small to detect, but the concept remains of interest as the only unambiguous example of reactionless motion in mainstream physics.
https://en.wikipedia.org/wiki/Reactionless_drive#Movement_without_thrust




By my understanding, it works through the fact the gravitational field is a gradient (slightly different in different positions, e.g. weaker further away from the source.). By shaping the body so that the center of mass of that body is in a different location where the gravitational field is stronger or weaker and then going back to the original shape, the body would have changed location ever so slightly compared to the previous location. This, however, does not impart any momentum on the body, as the force necessary to take on the shape is exactly the same force as the force to go back to the original shape (That's how I understood the theory, but the more I think about it, the forces required to change shapes should be different in different gravities.).



Here's a question on stackexchange on the topic: https://physics.stackexchange.com/questions/886/swimming-in-spacetime-apparent-conserved-quantity-violation
Here is a paper on the topic: http://science.sciencemag.org/content/299/5614/1865






share|improve this answer









$endgroup$




















    4












    $begingroup$

    Perhaps gravity, like that of the space probes. Or they could wear suits of magnets that attract or repel a planets or asteroids magnetic field. If they are some kind of animal perhaps they have a developed gland for an electrical field like an eel. Another idea is that they detect and capture neutrinos from the photons from stars and use them as a connective tissue or string. There are probably many different ideas of how they move because I’m pretty sure they cannot sit still in space. I would probably go with some type of jet stream orbit, and here is where their dust food would accumulate anyway.






    share|improve this answer









    $endgroup$












    • $begingroup$
      Yes, some good ideas and they probably use all those methods. However I specifically want them to swim in addition to anything else, so propulsion by body movement is what I'm after.
      $endgroup$
      – chasly from UK
      Nov 13 '18 at 18:51


















    4












    $begingroup$

    They are not actually swimming - their extremities are actually EM drives which allow for reactionless motion.






    share|improve this answer









    $endgroup$








    • 2




      $begingroup$
      You mean the Impossible Drive, which has never been shown to work in theory or in practice?
      $endgroup$
      – Nuclear Wang
      Nov 13 '18 at 18:51






    • 2




      $begingroup$
      EM Drives have no basis in science. It's like saying that magic exist because it was not proved not to exist.
      $endgroup$
      – Antzi
      Nov 14 '18 at 3:12






    • 2




      $begingroup$
      @Onyz All experiments showing thrust have either been irreproducible, flawed in design, or both.
      $endgroup$
      – Nuclear Wang
      Nov 14 '18 at 14:35






    • 1




      $begingroup$
      @Onyz Scott Manley did an update a few months ago, and still nothing, unfortunately
      $endgroup$
      – Aaron F
      Nov 14 '18 at 15:14






    • 5




      $begingroup$
      @Eth "breaking some of the most fundamental laws of physics" and even "would topple our entire understanding of science overnight" are not reasons to suggest that something does not work as advertised. If it works as advertised, then "physics is wrong" is a better answer than "that thing is not doing what we see it doing". The reason that the drive can be disbelieved is wholly and entirely different: it can be disbelieved because it has not been shown to work as advertised, and that is the primary reason.
      $endgroup$
      – Aaron
      Nov 14 '18 at 17:12


















    4












    $begingroup$

    Sorry but you will never be able to swim in there. One m3 of "space" will weight in order 10-21 to 10-25 kg. Your ordinary air is 1.225kg/m3. There is like 1019 grains of sand on Earth. You will destroy your body or run out of food/energy way before you produce some decent speed. Maybe use something like an Ion engine.






    share|improve this answer











    $endgroup$




















      3












      $begingroup$

      I can think of two methods of moving through space:




      1. Gravity assist—the creatures are highly attuned to gravity wells and know how to position themselves perfectly to slingshot around the universe.


      2. Pooping near the speed of light. If you just shoot a tiny poop at 0.999999C, then the energy is ridiculous enough for the "equal and opposite reaction" of Newton's laws to push your creature forward very well indeed.

      Actually moving fins against particles in space wont do enough, unless your creature weighs ~1 gram (but that's kinda silly, we'd be moving into fiction from here)



      Ok one more bonus point:




      1. Super-giant-colossal squid: This is similar to gravity assist, but with more control—tentacles are galaxy-sized in length, and at the end of them is where most of the creature's weight is. By moving the tentacles around, they can shift their weight around closer and farther from various sources of gravity.





      share|improve this answer











      $endgroup$








      • 1




        $begingroup$
        You mean that the momentum is ridiculous enough, energy is not whats referred to in the third of newtons laws. Though very good answer other than that
        $endgroup$
        – Ummdustry
        Nov 14 '18 at 13:12










      • $begingroup$
        The tentacles wouldn’t have to be that big. They could merely be lightyears across and move from star to star by ‘reaching out’ in the direction it wants to go, loping lazily from star to star.
        $endgroup$
        – Joe Bloggs
        Nov 14 '18 at 14:01










      • $begingroup$
        @Ummdustry It's technically correct, though; you need masses of energy to do that.
        $endgroup$
        – wizzwizz4
        Nov 14 '18 at 18:13


















      2












      $begingroup$

      They are not actually swimming.



      They are moving extremities in order to manipulate angular momentum. This allows them to change orientation.



      See for example https://en.wikipedia.org/wiki/Falling_cat_problem






      share|improve this answer









      $endgroup$




















        2












        $begingroup$

        Creature can emit light (or other form of radiation) from its skin cells, e.g. like light bugs or glowing sea creatures.



        Emitting light pushes the creature in opposite direction.



        Each cell can only emit light for a brief period of time. It takes time to prelenish the chemicals involved, and keeping the light on for too long will cook the cell.



        So as the creature sends a wave through its body, it activates lights on the part of the wave that is facing backward. It could probably achieve same effect by spinning, but either it gets dizzy, or it wants to keep its head pointed forward.






        share|improve this answer









        $endgroup$












        • $begingroup$
          That's a very novel solution! Although not strictly swimming, the movement would certainly look like it and the light-show would be amazing to watch.
          $endgroup$
          – chasly from UK
          Nov 14 '18 at 17:55



















        1












        $begingroup$

        Strictly speaking, with the size you have provided and the requirement of "Swimming" (Propulsion through self movement), what you are asking for is impossible. Swimming on our planet only works because we have things of sufficient density to displace to move ourselves forwards. Solar sails don't fit your criteria because they aren't active transportation. There are no active methods that you can use in the vacuum of space to propel yourself with no give/take.



        What I would recommend is making this amoeba/whale like a balloon. A thin amoeba shell over its preferred source of food and using the byproducts of digestion to propel slightly. It might take 100000000000 years to reach the next food source at sufficiently low speeds but you would eventually make it to something. maybe.



        The things that would work against you at this stage are :



        A. Gravity. It would take quite a bit of speed to get out of the solar systems and the eventual pull of the suns.



        B. Debris. It may be far and between but there is a good chance of eventually being punctured by debris given the distance and time.






        share|improve this answer









        $endgroup$












        • $begingroup$
          100000000000 years to reach the next food source is no problem. They simply go into suspended animation for as long as it takes. They are woken when warmed up by a star or by meeting food particles (interstellar dust).
          $endgroup$
          – chasly from UK
          Nov 14 '18 at 0:26











        • $begingroup$
          100b is ok? It is like 7 times age of the Universe.
          $endgroup$
          – Artemijs Danilovs
          Nov 14 '18 at 1:18










        • $begingroup$
          That is some pretty magical hibernation. technically, if the whale heads in the wrong direction it would never reach the next celestial body due to universal expansion.
          $endgroup$
          – IT Alex
          Nov 14 '18 at 12:58


















        1












        $begingroup$

        So what discoveries in physics since Newton will enable them to move this way?



        NONE



        But don't worry, I got you covered.



        Your space whale has a huge mouth and baleens that can collect the interstellar particules. Theses are then stocked as food and propellant.



        Your whale has a special organ that can expel any matter at very high speeds (read a ion engine). This organ needs electricity to function.



        That's where the huge flippers come into play. The space whale start moving its gigantic flippers and using a dedicated organ, stop them almost instantly when they reach top speed.



        The moment of inertia energy is converted into electricity used by the organ that actually propel your whale.



        Hence, the whale move by pulse and appear to swim into space, leaving a trail of space dust behind it. It goes into hibernation for long periods of time, in order to get enough propellant to accumulate in its mouth.






        share|improve this answer









        $endgroup$




















          1












          $begingroup$

          They are swimming in space. It's just that their local space is not as much of a near vacuum as our solar system or most of the interstellar medium. The local space happens to be the denser sections of a large, heavy accretion disk that has yet to fuse into a star or form planets. Or they swim when they pass through a binary star system, one star eating the other, with matter streaming from one point to the other giving them enough high speed, low density material to move against.



          If you're attached to them being interstellar, perhaps they return to the "stream" to mate, and the dancing is a ritual toward that goal, forming eddies and whorls that they can sense and find each other. Add "seeking love" and you satisfy your thematic requirements that they dance, and that the act can been seen as beautiful.






          share|improve this answer









          $endgroup$




















            1












            $begingroup$

            The 'swimming' motions are actually just used to pump whatever reaction-mass through the creature's body, expelling out the 'rear'.






            share|improve this answer









            $endgroup$




















              1












              $begingroup$

              It is obvious that they will not be the size of a solar system.



              This means they cannot have fins large enough to be useful in the vacuum of space and cannot swim that way.



              But if they had a naturally occurring powerful em field projected from their flippers perhaps they could use that em field to act as an invisible fin. Using it to push the atoms in space Around to propel this leviathan through space.



              How they are able to generate an em field large enough to effect a sufficient mass to use as propellant through space I leave to you.



              But it would work.






              share|improve this answer











              $endgroup$




















                0












                $begingroup$

                How about ion thrusters?



                The creatures inhale and store relatively small amount of xenon and due to internal biology(the wings are solar collectors?) produce enough electricity to to shoot xenon ions through some orifice in their body to produce thrust. I don't have the number on hand, but I understand that xenon ion thrusters are extremely efficient but they have very low acceleration



                While this isn't "swimming", the combination of wings (for solar) and thrust might look enough like swimming/flying to the casual observer






                share|improve this answer











                $endgroup$












                • $begingroup$
                  This does not address the OP's request for a mechanical way to "swim" in space.
                  $endgroup$
                  – anon
                  Nov 14 '18 at 14:59










                • $begingroup$
                  @anon is this better?
                  $endgroup$
                  – Nullman
                  Nov 14 '18 at 15:02


















                0












                $begingroup$

                If you want to get more speculative, they swim through the "Luminiferous aether".



                The aether was a theoretical medium through which light was thought to propagate, thus explaining its wave-like nature. The theory was discarded as ever-more-sensitive experiments failed to detect any ripples in the apparent speed of light caused by the momentum of Earth not matching the momentum of the aether...



                Of course, we have since discovered that all of the elementary particles of matter also travel as waves. (This is how electron microscopes work) The wavelength merely decreases as the particles get larger until it is too short for us to detect. Which would offer a theoretical alternative explanation for why the experiments detected no variation between light and matter due to aetheric ripples: The test equipment itself is also an aetheric ripple and therefore there can never be a difference in medium velocity between light and matter.



                Your creatures have found some way to push against the aether itself. (Most likely the creatures are actually the ripples caused by the actions of a higher-level entity that is not entirely perceptible to our senses.) This would allow them to "swim" through seemingly empty (ripple-free) space.



                There are a few potential side-effects to this:



                Such creatures could be a source of extra-universal energy to keep the universe running (similar to the theorized, but never discovered "white hole").



                Such creatures would be theoretically capable of exceeding the speed of light.



                Such creatures would likely be harmed only by (from our point of view) exceedingly violent events. (If all matter is ripples in the aether, they are the water-bug swimming on the surface. It would take some really big ripples to drown them.)



                Such creatures might appear spontaneously in relatively calm places and disappear in turbulent places. (The swimmers get out of the pool when the water gets too churned up.)






                share|improve this answer









                $endgroup$












                  Your Answer





                  StackExchange.ifUsing("editor", function ()
                  return StackExchange.using("mathjaxEditing", function ()
                  StackExchange.MarkdownEditor.creationCallbacks.add(function (editor, postfix)
                  StackExchange.mathjaxEditing.prepareWmdForMathJax(editor, postfix, [["$", "$"], ["\\(","\\)"]]);
                  );
                  );
                  , "mathjax-editing");

                  StackExchange.ready(function()
                  var channelOptions =
                  tags: "".split(" "),
                  id: "579"
                  ;
                  initTagRenderer("".split(" "), "".split(" "), channelOptions);

                  StackExchange.using("externalEditor", function()
                  // Have to fire editor after snippets, if snippets enabled
                  if (StackExchange.settings.snippets.snippetsEnabled)
                  StackExchange.using("snippets", function()
                  createEditor();
                  );

                  else
                  createEditor();

                  );

                  function createEditor()
                  StackExchange.prepareEditor(
                  heartbeatType: 'answer',
                  autoActivateHeartbeat: false,
                  convertImagesToLinks: false,
                  noModals: true,
                  showLowRepImageUploadWarning: true,
                  reputationToPostImages: null,
                  bindNavPrevention: true,
                  postfix: "",
                  imageUploader:
                  brandingHtml: "Powered by u003ca class="icon-imgur-white" href="https://imgur.com/"u003eu003c/au003e",
                  contentPolicyHtml: "User contributions licensed under u003ca href="https://creativecommons.org/licenses/by-sa/3.0/"u003ecc by-sa 3.0 with attribution requiredu003c/au003e u003ca href="https://stackoverflow.com/legal/content-policy"u003e(content policy)u003c/au003e",
                  allowUrls: true
                  ,
                  noCode: true, onDemand: true,
                  discardSelector: ".discard-answer"
                  ,immediatelyShowMarkdownHelp:true
                  );



                  );













                  draft saved

                  draft discarded


















                  StackExchange.ready(
                  function ()
                  StackExchange.openid.initPostLogin('.new-post-login', 'https%3a%2f%2fworldbuilding.stackexchange.com%2fquestions%2f130160%2fswimming-in-space-what-is-the-mechanism%23new-answer', 'question_page');

                  );

                  Post as a guest















                  Required, but never shown

























                  17 Answers
                  17






                  active

                  oldest

                  votes








                  17 Answers
                  17






                  active

                  oldest

                  votes









                  active

                  oldest

                  votes






                  active

                  oldest

                  votes









                  24












                  $begingroup$

                  Space is not perfectly empty. There are some sparse atoms and particles, which, under the right conditions, can be used to exert a force and use Newton 3rd law.



                  If you want to use fluid dynamic forces in space, you just need big wings. How big?



                  This answer gives an estimate:




                  Because of the low density of particles in the interstellar medium, a normal-sized wing will not act as an airfoil. It will be more like bouncing tennis balls off the wing every so often.



                  The Knudsen number quantifies when fluid dynamics takes over: Kn = mean_free_path / airfoil_length_scale. Knudsen numbers greater than about 10 are in the regime of ballistic collisions, rather than fluid flow. The mean free path in the interstellar medium is huge (about 70 astronomical units). So to get fluid dynamics, you'd need an airfoil of size more than the Sun–Jupiter distance.







                  share|improve this answer









                  $endgroup$








                  • 1




                    $begingroup$
                    I've added something about the size of the creatures. Is an air-foil actually necessary? That's why I called it swimming rather than flying. For example I don't believe an octopus or a jellyfish use air-foils to move through water. (or do they?)
                    $endgroup$
                    – chasly from UK
                    Nov 13 '18 at 19:46











                  • $begingroup$
                    @chaslyfromUK, an octopus uses reaction (which you covered)
                    $endgroup$
                    – L.Dutch
                    Nov 13 '18 at 19:50






                  • 2




                    $begingroup$
                    Reaction against faeces yes. What about gulping interstellar gas and passing it through the body without absorption? Will that work?
                    $endgroup$
                    – chasly from UK
                    Nov 13 '18 at 19:59






                  • 2




                    $begingroup$
                    Ballistic is fine for pushing on matter with large flat panels, if you can move them as fast as you want to travel (relative to the medium). You can rotate them as you pull them forward to reduce their surface area to near-0, and to max for the downstroke. Or a spinning fan could maybe push on matter. Neither are very practical for a large living creature, and the low density just makes it all around impractical, but being in the ballistic regime is not a show stopper on its own. You could "swim" through a field of floating heavy tennis balls by on average pushing them backwards.
                    $endgroup$
                    – Peter Cordes
                    Nov 14 '18 at 0:47






                  • 1




                    $begingroup$
                    @PeterCordes: a matrix of wings might make more sense. I can imagine huge feathery fractaline cilia with billions of ‘rapidly’ moving wings all arrayed in a huge panel to provide thrust.
                    $endgroup$
                    – Joe Bloggs
                    Nov 14 '18 at 13:55















                  24












                  $begingroup$

                  Space is not perfectly empty. There are some sparse atoms and particles, which, under the right conditions, can be used to exert a force and use Newton 3rd law.



                  If you want to use fluid dynamic forces in space, you just need big wings. How big?



                  This answer gives an estimate:




                  Because of the low density of particles in the interstellar medium, a normal-sized wing will not act as an airfoil. It will be more like bouncing tennis balls off the wing every so often.



                  The Knudsen number quantifies when fluid dynamics takes over: Kn = mean_free_path / airfoil_length_scale. Knudsen numbers greater than about 10 are in the regime of ballistic collisions, rather than fluid flow. The mean free path in the interstellar medium is huge (about 70 astronomical units). So to get fluid dynamics, you'd need an airfoil of size more than the Sun–Jupiter distance.







                  share|improve this answer









                  $endgroup$








                  • 1




                    $begingroup$
                    I've added something about the size of the creatures. Is an air-foil actually necessary? That's why I called it swimming rather than flying. For example I don't believe an octopus or a jellyfish use air-foils to move through water. (or do they?)
                    $endgroup$
                    – chasly from UK
                    Nov 13 '18 at 19:46











                  • $begingroup$
                    @chaslyfromUK, an octopus uses reaction (which you covered)
                    $endgroup$
                    – L.Dutch
                    Nov 13 '18 at 19:50






                  • 2




                    $begingroup$
                    Reaction against faeces yes. What about gulping interstellar gas and passing it through the body without absorption? Will that work?
                    $endgroup$
                    – chasly from UK
                    Nov 13 '18 at 19:59






                  • 2




                    $begingroup$
                    Ballistic is fine for pushing on matter with large flat panels, if you can move them as fast as you want to travel (relative to the medium). You can rotate them as you pull them forward to reduce their surface area to near-0, and to max for the downstroke. Or a spinning fan could maybe push on matter. Neither are very practical for a large living creature, and the low density just makes it all around impractical, but being in the ballistic regime is not a show stopper on its own. You could "swim" through a field of floating heavy tennis balls by on average pushing them backwards.
                    $endgroup$
                    – Peter Cordes
                    Nov 14 '18 at 0:47






                  • 1




                    $begingroup$
                    @PeterCordes: a matrix of wings might make more sense. I can imagine huge feathery fractaline cilia with billions of ‘rapidly’ moving wings all arrayed in a huge panel to provide thrust.
                    $endgroup$
                    – Joe Bloggs
                    Nov 14 '18 at 13:55













                  24












                  24








                  24





                  $begingroup$

                  Space is not perfectly empty. There are some sparse atoms and particles, which, under the right conditions, can be used to exert a force and use Newton 3rd law.



                  If you want to use fluid dynamic forces in space, you just need big wings. How big?



                  This answer gives an estimate:




                  Because of the low density of particles in the interstellar medium, a normal-sized wing will not act as an airfoil. It will be more like bouncing tennis balls off the wing every so often.



                  The Knudsen number quantifies when fluid dynamics takes over: Kn = mean_free_path / airfoil_length_scale. Knudsen numbers greater than about 10 are in the regime of ballistic collisions, rather than fluid flow. The mean free path in the interstellar medium is huge (about 70 astronomical units). So to get fluid dynamics, you'd need an airfoil of size more than the Sun–Jupiter distance.







                  share|improve this answer









                  $endgroup$



                  Space is not perfectly empty. There are some sparse atoms and particles, which, under the right conditions, can be used to exert a force and use Newton 3rd law.



                  If you want to use fluid dynamic forces in space, you just need big wings. How big?



                  This answer gives an estimate:




                  Because of the low density of particles in the interstellar medium, a normal-sized wing will not act as an airfoil. It will be more like bouncing tennis balls off the wing every so often.



                  The Knudsen number quantifies when fluid dynamics takes over: Kn = mean_free_path / airfoil_length_scale. Knudsen numbers greater than about 10 are in the regime of ballistic collisions, rather than fluid flow. The mean free path in the interstellar medium is huge (about 70 astronomical units). So to get fluid dynamics, you'd need an airfoil of size more than the Sun–Jupiter distance.








                  share|improve this answer












                  share|improve this answer



                  share|improve this answer










                  answered Nov 13 '18 at 18:23









                  L.DutchL.Dutch

                  80.4k26192391




                  80.4k26192391







                  • 1




                    $begingroup$
                    I've added something about the size of the creatures. Is an air-foil actually necessary? That's why I called it swimming rather than flying. For example I don't believe an octopus or a jellyfish use air-foils to move through water. (or do they?)
                    $endgroup$
                    – chasly from UK
                    Nov 13 '18 at 19:46











                  • $begingroup$
                    @chaslyfromUK, an octopus uses reaction (which you covered)
                    $endgroup$
                    – L.Dutch
                    Nov 13 '18 at 19:50






                  • 2




                    $begingroup$
                    Reaction against faeces yes. What about gulping interstellar gas and passing it through the body without absorption? Will that work?
                    $endgroup$
                    – chasly from UK
                    Nov 13 '18 at 19:59






                  • 2




                    $begingroup$
                    Ballistic is fine for pushing on matter with large flat panels, if you can move them as fast as you want to travel (relative to the medium). You can rotate them as you pull them forward to reduce their surface area to near-0, and to max for the downstroke. Or a spinning fan could maybe push on matter. Neither are very practical for a large living creature, and the low density just makes it all around impractical, but being in the ballistic regime is not a show stopper on its own. You could "swim" through a field of floating heavy tennis balls by on average pushing them backwards.
                    $endgroup$
                    – Peter Cordes
                    Nov 14 '18 at 0:47






                  • 1




                    $begingroup$
                    @PeterCordes: a matrix of wings might make more sense. I can imagine huge feathery fractaline cilia with billions of ‘rapidly’ moving wings all arrayed in a huge panel to provide thrust.
                    $endgroup$
                    – Joe Bloggs
                    Nov 14 '18 at 13:55












                  • 1




                    $begingroup$
                    I've added something about the size of the creatures. Is an air-foil actually necessary? That's why I called it swimming rather than flying. For example I don't believe an octopus or a jellyfish use air-foils to move through water. (or do they?)
                    $endgroup$
                    – chasly from UK
                    Nov 13 '18 at 19:46











                  • $begingroup$
                    @chaslyfromUK, an octopus uses reaction (which you covered)
                    $endgroup$
                    – L.Dutch
                    Nov 13 '18 at 19:50






                  • 2




                    $begingroup$
                    Reaction against faeces yes. What about gulping interstellar gas and passing it through the body without absorption? Will that work?
                    $endgroup$
                    – chasly from UK
                    Nov 13 '18 at 19:59






                  • 2




                    $begingroup$
                    Ballistic is fine for pushing on matter with large flat panels, if you can move them as fast as you want to travel (relative to the medium). You can rotate them as you pull them forward to reduce their surface area to near-0, and to max for the downstroke. Or a spinning fan could maybe push on matter. Neither are very practical for a large living creature, and the low density just makes it all around impractical, but being in the ballistic regime is not a show stopper on its own. You could "swim" through a field of floating heavy tennis balls by on average pushing them backwards.
                    $endgroup$
                    – Peter Cordes
                    Nov 14 '18 at 0:47






                  • 1




                    $begingroup$
                    @PeterCordes: a matrix of wings might make more sense. I can imagine huge feathery fractaline cilia with billions of ‘rapidly’ moving wings all arrayed in a huge panel to provide thrust.
                    $endgroup$
                    – Joe Bloggs
                    Nov 14 '18 at 13:55







                  1




                  1




                  $begingroup$
                  I've added something about the size of the creatures. Is an air-foil actually necessary? That's why I called it swimming rather than flying. For example I don't believe an octopus or a jellyfish use air-foils to move through water. (or do they?)
                  $endgroup$
                  – chasly from UK
                  Nov 13 '18 at 19:46





                  $begingroup$
                  I've added something about the size of the creatures. Is an air-foil actually necessary? That's why I called it swimming rather than flying. For example I don't believe an octopus or a jellyfish use air-foils to move through water. (or do they?)
                  $endgroup$
                  – chasly from UK
                  Nov 13 '18 at 19:46













                  $begingroup$
                  @chaslyfromUK, an octopus uses reaction (which you covered)
                  $endgroup$
                  – L.Dutch
                  Nov 13 '18 at 19:50




                  $begingroup$
                  @chaslyfromUK, an octopus uses reaction (which you covered)
                  $endgroup$
                  – L.Dutch
                  Nov 13 '18 at 19:50




                  2




                  2




                  $begingroup$
                  Reaction against faeces yes. What about gulping interstellar gas and passing it through the body without absorption? Will that work?
                  $endgroup$
                  – chasly from UK
                  Nov 13 '18 at 19:59




                  $begingroup$
                  Reaction against faeces yes. What about gulping interstellar gas and passing it through the body without absorption? Will that work?
                  $endgroup$
                  – chasly from UK
                  Nov 13 '18 at 19:59




                  2




                  2




                  $begingroup$
                  Ballistic is fine for pushing on matter with large flat panels, if you can move them as fast as you want to travel (relative to the medium). You can rotate them as you pull them forward to reduce their surface area to near-0, and to max for the downstroke. Or a spinning fan could maybe push on matter. Neither are very practical for a large living creature, and the low density just makes it all around impractical, but being in the ballistic regime is not a show stopper on its own. You could "swim" through a field of floating heavy tennis balls by on average pushing them backwards.
                  $endgroup$
                  – Peter Cordes
                  Nov 14 '18 at 0:47




                  $begingroup$
                  Ballistic is fine for pushing on matter with large flat panels, if you can move them as fast as you want to travel (relative to the medium). You can rotate them as you pull them forward to reduce their surface area to near-0, and to max for the downstroke. Or a spinning fan could maybe push on matter. Neither are very practical for a large living creature, and the low density just makes it all around impractical, but being in the ballistic regime is not a show stopper on its own. You could "swim" through a field of floating heavy tennis balls by on average pushing them backwards.
                  $endgroup$
                  – Peter Cordes
                  Nov 14 '18 at 0:47




                  1




                  1




                  $begingroup$
                  @PeterCordes: a matrix of wings might make more sense. I can imagine huge feathery fractaline cilia with billions of ‘rapidly’ moving wings all arrayed in a huge panel to provide thrust.
                  $endgroup$
                  – Joe Bloggs
                  Nov 14 '18 at 13:55




                  $begingroup$
                  @PeterCordes: a matrix of wings might make more sense. I can imagine huge feathery fractaline cilia with billions of ‘rapidly’ moving wings all arrayed in a huge panel to provide thrust.
                  $endgroup$
                  – Joe Bloggs
                  Nov 14 '18 at 13:55











                  12












                  $begingroup$

                  Negative mass.



                  Each of your creatures contains a precious nugget of negative mass. When one of these creatures reproduces, the offspring are each given a fragment of the nugget. As they travel the universe, each creature accumulates any additional negative mass encountered and adds it to the nugget. When a creature dies, the nugget is claimed by its kin.



                  Negative mass can be used to propel. The larger and more unmassive this chunk is, the more useful it is for propulsion.



                  https://en.wikipedia.org/wiki/Negative_mass




                  For two positive masses, nothing changes and there is a gravitational
                  pull on each other causing an attraction. Two negative masses would
                  repel because of their negative inertial masses. For different signs
                  however, there is a push that repels the positive mass from the
                  negative mass, and a pull that attracts the negative mass towards the
                  positive one at the same time.



                  Hence Bondi pointed out that two objects of equal and opposite mass
                  would produce a constant acceleration of the system towards the
                  positive-mass object,[4] an effect called "runaway motion"...



                  Such a couple of objects would accelerate without limit (except
                  relativistic one); however, the total mass, momentum and energy of the
                  system would remain 0.




                  The nugget would be kept in the center of the body. When needing propulsion, the nugget would be moved to the end of an appendage. There it would repel the main body of the creature and be attracted at the same time. The same principle could be used to brake or steer by moving the nugget to a different position.






                  share|improve this answer









                  $endgroup$












                  • $begingroup$
                    I remember reading about negative mass in an old edition of New Scientist. I like this idea. The "swimming" motions could very well be how the creature resolves steering and acceleration changes.
                    $endgroup$
                    – Baldrickk
                    Nov 14 '18 at 11:05










                  • $begingroup$
                    This sounds like fiction; am I correct in assuming it is? That's fine if so, obviously, but I just wanted to make sure. Simply stating "the total mass, momentum and energy of the system would remain 0" seems to assume only its own reference frame, as using 2 of these things would allow for perpetual motion machines with a net gain in energy.
                    $endgroup$
                    – Aaron
                    Nov 14 '18 at 17:04






                  • 2




                    $begingroup$
                    The maths of negative math is a known and real thing. It's never been shown to exist and as you point out has some.....problems... but so far as I know it hasn't been proven impossible either.
                    $endgroup$
                    – Tim B
                    Nov 14 '18 at 17:31










                  • $begingroup$
                    @Aaron - follow the link and then keep going, if this interests you. The infamous Alcubierre drive relies on negative matter to work.
                    $endgroup$
                    – Willk
                    Nov 14 '18 at 17:52






                  • 1




                    $begingroup$
                    Unless the negative mass is equal to the positive mass, it doesn't matter where the mass is located in the body. The total net effect is to lower the overall mass.This makes other means of propulsion more effective, but gives no net propulsion itself. The force it generates is cancelled by the force used to hold it. The acceleration mentioned in the article only occurs when the negative and positive mass are exactly equal in magnitude. Otherwise, negative mass can only be used for acceleration by expelling it and allowing it to move off in the opposite direction.
                    $endgroup$
                    – Paul Sinclair
                    Nov 14 '18 at 18:10
















                  12












                  $begingroup$

                  Negative mass.



                  Each of your creatures contains a precious nugget of negative mass. When one of these creatures reproduces, the offspring are each given a fragment of the nugget. As they travel the universe, each creature accumulates any additional negative mass encountered and adds it to the nugget. When a creature dies, the nugget is claimed by its kin.



                  Negative mass can be used to propel. The larger and more unmassive this chunk is, the more useful it is for propulsion.



                  https://en.wikipedia.org/wiki/Negative_mass




                  For two positive masses, nothing changes and there is a gravitational
                  pull on each other causing an attraction. Two negative masses would
                  repel because of their negative inertial masses. For different signs
                  however, there is a push that repels the positive mass from the
                  negative mass, and a pull that attracts the negative mass towards the
                  positive one at the same time.



                  Hence Bondi pointed out that two objects of equal and opposite mass
                  would produce a constant acceleration of the system towards the
                  positive-mass object,[4] an effect called "runaway motion"...



                  Such a couple of objects would accelerate without limit (except
                  relativistic one); however, the total mass, momentum and energy of the
                  system would remain 0.




                  The nugget would be kept in the center of the body. When needing propulsion, the nugget would be moved to the end of an appendage. There it would repel the main body of the creature and be attracted at the same time. The same principle could be used to brake or steer by moving the nugget to a different position.






                  share|improve this answer









                  $endgroup$












                  • $begingroup$
                    I remember reading about negative mass in an old edition of New Scientist. I like this idea. The "swimming" motions could very well be how the creature resolves steering and acceleration changes.
                    $endgroup$
                    – Baldrickk
                    Nov 14 '18 at 11:05










                  • $begingroup$
                    This sounds like fiction; am I correct in assuming it is? That's fine if so, obviously, but I just wanted to make sure. Simply stating "the total mass, momentum and energy of the system would remain 0" seems to assume only its own reference frame, as using 2 of these things would allow for perpetual motion machines with a net gain in energy.
                    $endgroup$
                    – Aaron
                    Nov 14 '18 at 17:04






                  • 2




                    $begingroup$
                    The maths of negative math is a known and real thing. It's never been shown to exist and as you point out has some.....problems... but so far as I know it hasn't been proven impossible either.
                    $endgroup$
                    – Tim B
                    Nov 14 '18 at 17:31










                  • $begingroup$
                    @Aaron - follow the link and then keep going, if this interests you. The infamous Alcubierre drive relies on negative matter to work.
                    $endgroup$
                    – Willk
                    Nov 14 '18 at 17:52






                  • 1




                    $begingroup$
                    Unless the negative mass is equal to the positive mass, it doesn't matter where the mass is located in the body. The total net effect is to lower the overall mass.This makes other means of propulsion more effective, but gives no net propulsion itself. The force it generates is cancelled by the force used to hold it. The acceleration mentioned in the article only occurs when the negative and positive mass are exactly equal in magnitude. Otherwise, negative mass can only be used for acceleration by expelling it and allowing it to move off in the opposite direction.
                    $endgroup$
                    – Paul Sinclair
                    Nov 14 '18 at 18:10














                  12












                  12








                  12





                  $begingroup$

                  Negative mass.



                  Each of your creatures contains a precious nugget of negative mass. When one of these creatures reproduces, the offspring are each given a fragment of the nugget. As they travel the universe, each creature accumulates any additional negative mass encountered and adds it to the nugget. When a creature dies, the nugget is claimed by its kin.



                  Negative mass can be used to propel. The larger and more unmassive this chunk is, the more useful it is for propulsion.



                  https://en.wikipedia.org/wiki/Negative_mass




                  For two positive masses, nothing changes and there is a gravitational
                  pull on each other causing an attraction. Two negative masses would
                  repel because of their negative inertial masses. For different signs
                  however, there is a push that repels the positive mass from the
                  negative mass, and a pull that attracts the negative mass towards the
                  positive one at the same time.



                  Hence Bondi pointed out that two objects of equal and opposite mass
                  would produce a constant acceleration of the system towards the
                  positive-mass object,[4] an effect called "runaway motion"...



                  Such a couple of objects would accelerate without limit (except
                  relativistic one); however, the total mass, momentum and energy of the
                  system would remain 0.




                  The nugget would be kept in the center of the body. When needing propulsion, the nugget would be moved to the end of an appendage. There it would repel the main body of the creature and be attracted at the same time. The same principle could be used to brake or steer by moving the nugget to a different position.






                  share|improve this answer









                  $endgroup$



                  Negative mass.



                  Each of your creatures contains a precious nugget of negative mass. When one of these creatures reproduces, the offspring are each given a fragment of the nugget. As they travel the universe, each creature accumulates any additional negative mass encountered and adds it to the nugget. When a creature dies, the nugget is claimed by its kin.



                  Negative mass can be used to propel. The larger and more unmassive this chunk is, the more useful it is for propulsion.



                  https://en.wikipedia.org/wiki/Negative_mass




                  For two positive masses, nothing changes and there is a gravitational
                  pull on each other causing an attraction. Two negative masses would
                  repel because of their negative inertial masses. For different signs
                  however, there is a push that repels the positive mass from the
                  negative mass, and a pull that attracts the negative mass towards the
                  positive one at the same time.



                  Hence Bondi pointed out that two objects of equal and opposite mass
                  would produce a constant acceleration of the system towards the
                  positive-mass object,[4] an effect called "runaway motion"...



                  Such a couple of objects would accelerate without limit (except
                  relativistic one); however, the total mass, momentum and energy of the
                  system would remain 0.




                  The nugget would be kept in the center of the body. When needing propulsion, the nugget would be moved to the end of an appendage. There it would repel the main body of the creature and be attracted at the same time. The same principle could be used to brake or steer by moving the nugget to a different position.







                  share|improve this answer












                  share|improve this answer



                  share|improve this answer










                  answered Nov 13 '18 at 21:08









                  WillkWillk

                  104k25197440




                  104k25197440











                  • $begingroup$
                    I remember reading about negative mass in an old edition of New Scientist. I like this idea. The "swimming" motions could very well be how the creature resolves steering and acceleration changes.
                    $endgroup$
                    – Baldrickk
                    Nov 14 '18 at 11:05










                  • $begingroup$
                    This sounds like fiction; am I correct in assuming it is? That's fine if so, obviously, but I just wanted to make sure. Simply stating "the total mass, momentum and energy of the system would remain 0" seems to assume only its own reference frame, as using 2 of these things would allow for perpetual motion machines with a net gain in energy.
                    $endgroup$
                    – Aaron
                    Nov 14 '18 at 17:04






                  • 2




                    $begingroup$
                    The maths of negative math is a known and real thing. It's never been shown to exist and as you point out has some.....problems... but so far as I know it hasn't been proven impossible either.
                    $endgroup$
                    – Tim B
                    Nov 14 '18 at 17:31










                  • $begingroup$
                    @Aaron - follow the link and then keep going, if this interests you. The infamous Alcubierre drive relies on negative matter to work.
                    $endgroup$
                    – Willk
                    Nov 14 '18 at 17:52






                  • 1




                    $begingroup$
                    Unless the negative mass is equal to the positive mass, it doesn't matter where the mass is located in the body. The total net effect is to lower the overall mass.This makes other means of propulsion more effective, but gives no net propulsion itself. The force it generates is cancelled by the force used to hold it. The acceleration mentioned in the article only occurs when the negative and positive mass are exactly equal in magnitude. Otherwise, negative mass can only be used for acceleration by expelling it and allowing it to move off in the opposite direction.
                    $endgroup$
                    – Paul Sinclair
                    Nov 14 '18 at 18:10

















                  • $begingroup$
                    I remember reading about negative mass in an old edition of New Scientist. I like this idea. The "swimming" motions could very well be how the creature resolves steering and acceleration changes.
                    $endgroup$
                    – Baldrickk
                    Nov 14 '18 at 11:05










                  • $begingroup$
                    This sounds like fiction; am I correct in assuming it is? That's fine if so, obviously, but I just wanted to make sure. Simply stating "the total mass, momentum and energy of the system would remain 0" seems to assume only its own reference frame, as using 2 of these things would allow for perpetual motion machines with a net gain in energy.
                    $endgroup$
                    – Aaron
                    Nov 14 '18 at 17:04






                  • 2




                    $begingroup$
                    The maths of negative math is a known and real thing. It's never been shown to exist and as you point out has some.....problems... but so far as I know it hasn't been proven impossible either.
                    $endgroup$
                    – Tim B
                    Nov 14 '18 at 17:31










                  • $begingroup$
                    @Aaron - follow the link and then keep going, if this interests you. The infamous Alcubierre drive relies on negative matter to work.
                    $endgroup$
                    – Willk
                    Nov 14 '18 at 17:52






                  • 1




                    $begingroup$
                    Unless the negative mass is equal to the positive mass, it doesn't matter where the mass is located in the body. The total net effect is to lower the overall mass.This makes other means of propulsion more effective, but gives no net propulsion itself. The force it generates is cancelled by the force used to hold it. The acceleration mentioned in the article only occurs when the negative and positive mass are exactly equal in magnitude. Otherwise, negative mass can only be used for acceleration by expelling it and allowing it to move off in the opposite direction.
                    $endgroup$
                    – Paul Sinclair
                    Nov 14 '18 at 18:10
















                  $begingroup$
                  I remember reading about negative mass in an old edition of New Scientist. I like this idea. The "swimming" motions could very well be how the creature resolves steering and acceleration changes.
                  $endgroup$
                  – Baldrickk
                  Nov 14 '18 at 11:05




                  $begingroup$
                  I remember reading about negative mass in an old edition of New Scientist. I like this idea. The "swimming" motions could very well be how the creature resolves steering and acceleration changes.
                  $endgroup$
                  – Baldrickk
                  Nov 14 '18 at 11:05












                  $begingroup$
                  This sounds like fiction; am I correct in assuming it is? That's fine if so, obviously, but I just wanted to make sure. Simply stating "the total mass, momentum and energy of the system would remain 0" seems to assume only its own reference frame, as using 2 of these things would allow for perpetual motion machines with a net gain in energy.
                  $endgroup$
                  – Aaron
                  Nov 14 '18 at 17:04




                  $begingroup$
                  This sounds like fiction; am I correct in assuming it is? That's fine if so, obviously, but I just wanted to make sure. Simply stating "the total mass, momentum and energy of the system would remain 0" seems to assume only its own reference frame, as using 2 of these things would allow for perpetual motion machines with a net gain in energy.
                  $endgroup$
                  – Aaron
                  Nov 14 '18 at 17:04




                  2




                  2




                  $begingroup$
                  The maths of negative math is a known and real thing. It's never been shown to exist and as you point out has some.....problems... but so far as I know it hasn't been proven impossible either.
                  $endgroup$
                  – Tim B
                  Nov 14 '18 at 17:31




                  $begingroup$
                  The maths of negative math is a known and real thing. It's never been shown to exist and as you point out has some.....problems... but so far as I know it hasn't been proven impossible either.
                  $endgroup$
                  – Tim B
                  Nov 14 '18 at 17:31












                  $begingroup$
                  @Aaron - follow the link and then keep going, if this interests you. The infamous Alcubierre drive relies on negative matter to work.
                  $endgroup$
                  – Willk
                  Nov 14 '18 at 17:52




                  $begingroup$
                  @Aaron - follow the link and then keep going, if this interests you. The infamous Alcubierre drive relies on negative matter to work.
                  $endgroup$
                  – Willk
                  Nov 14 '18 at 17:52




                  1




                  1




                  $begingroup$
                  Unless the negative mass is equal to the positive mass, it doesn't matter where the mass is located in the body. The total net effect is to lower the overall mass.This makes other means of propulsion more effective, but gives no net propulsion itself. The force it generates is cancelled by the force used to hold it. The acceleration mentioned in the article only occurs when the negative and positive mass are exactly equal in magnitude. Otherwise, negative mass can only be used for acceleration by expelling it and allowing it to move off in the opposite direction.
                  $endgroup$
                  – Paul Sinclair
                  Nov 14 '18 at 18:10





                  $begingroup$
                  Unless the negative mass is equal to the positive mass, it doesn't matter where the mass is located in the body. The total net effect is to lower the overall mass.This makes other means of propulsion more effective, but gives no net propulsion itself. The force it generates is cancelled by the force used to hold it. The acceleration mentioned in the article only occurs when the negative and positive mass are exactly equal in magnitude. Otherwise, negative mass can only be used for acceleration by expelling it and allowing it to move off in the opposite direction.
                  $endgroup$
                  – Paul Sinclair
                  Nov 14 '18 at 18:10












                  11












                  $begingroup$

                  The creature is actually a giant bussard ramjet. It just looks like it's swimming, because the flaps are used to move particles from the surrounding area towards the body, where they are collected by polyp-like areas covering the body. They are then moved towards an organ that holds the particles until they are required for energy generation or used as propulsion matter. The body could also emit an EM field that moves particles closer to the body, so that the flaps can collect more of it.



                  The end result would be a movement similar to the breaststroke of a swimmer with a forward movement of the flaps close to the body to avoid pushing particles away and a wide backwards movement to collect a large volume of particles.



                  It's probably not scientifically viable, because the movement would most likely expand more energy than it could generate from what it collects, but it's at least science based. You could make the flaps solar energy collectors and have it hibernate during times where it's too far away from a sun to collect energy, if you want it's energy consumption to be slightly more realistic.






                  share|improve this answer











                  $endgroup$












                  • $begingroup$
                    L.Dutch covered the size of said ramjet in their answer. Not exactly, but the sale math applies I think. IANARS.
                    $endgroup$
                    – Draco18s
                    Nov 14 '18 at 16:51






                  • 1




                    $begingroup$
                    @Draco18s Nah, that's talking about different things. Bussard Ramjets use completely different principles and function at much more reasonable scales (although last I heard people thought they would probably work better as brakes than engines!)
                    $endgroup$
                    – Tim B
                    Nov 14 '18 at 17:22






                  • 1




                    $begingroup$
                    @TimB I haven't seen the science, and IANARS, so I'm just guessing. Point is, space is still ridiculously not-dense (the densest nebulae are less dense than the best laboratory vacuum chamber).
                    $endgroup$
                    – Draco18s
                    Nov 14 '18 at 17:28






                  • 1




                    $begingroup$
                    @Draco18s Wings use fluid dynamics, that's what the other answer covers. Bussard Ramjets use electromagnetic fields to gather interstellar medium to then use as fuel and propellant. It's a completely different mechanism.
                    $endgroup$
                    – Tim B
                    Nov 14 '18 at 17:30















                  11












                  $begingroup$

                  The creature is actually a giant bussard ramjet. It just looks like it's swimming, because the flaps are used to move particles from the surrounding area towards the body, where they are collected by polyp-like areas covering the body. They are then moved towards an organ that holds the particles until they are required for energy generation or used as propulsion matter. The body could also emit an EM field that moves particles closer to the body, so that the flaps can collect more of it.



                  The end result would be a movement similar to the breaststroke of a swimmer with a forward movement of the flaps close to the body to avoid pushing particles away and a wide backwards movement to collect a large volume of particles.



                  It's probably not scientifically viable, because the movement would most likely expand more energy than it could generate from what it collects, but it's at least science based. You could make the flaps solar energy collectors and have it hibernate during times where it's too far away from a sun to collect energy, if you want it's energy consumption to be slightly more realistic.






                  share|improve this answer











                  $endgroup$












                  • $begingroup$
                    L.Dutch covered the size of said ramjet in their answer. Not exactly, but the sale math applies I think. IANARS.
                    $endgroup$
                    – Draco18s
                    Nov 14 '18 at 16:51






                  • 1




                    $begingroup$
                    @Draco18s Nah, that's talking about different things. Bussard Ramjets use completely different principles and function at much more reasonable scales (although last I heard people thought they would probably work better as brakes than engines!)
                    $endgroup$
                    – Tim B
                    Nov 14 '18 at 17:22






                  • 1




                    $begingroup$
                    @TimB I haven't seen the science, and IANARS, so I'm just guessing. Point is, space is still ridiculously not-dense (the densest nebulae are less dense than the best laboratory vacuum chamber).
                    $endgroup$
                    – Draco18s
                    Nov 14 '18 at 17:28






                  • 1




                    $begingroup$
                    @Draco18s Wings use fluid dynamics, that's what the other answer covers. Bussard Ramjets use electromagnetic fields to gather interstellar medium to then use as fuel and propellant. It's a completely different mechanism.
                    $endgroup$
                    – Tim B
                    Nov 14 '18 at 17:30













                  11












                  11








                  11





                  $begingroup$

                  The creature is actually a giant bussard ramjet. It just looks like it's swimming, because the flaps are used to move particles from the surrounding area towards the body, where they are collected by polyp-like areas covering the body. They are then moved towards an organ that holds the particles until they are required for energy generation or used as propulsion matter. The body could also emit an EM field that moves particles closer to the body, so that the flaps can collect more of it.



                  The end result would be a movement similar to the breaststroke of a swimmer with a forward movement of the flaps close to the body to avoid pushing particles away and a wide backwards movement to collect a large volume of particles.



                  It's probably not scientifically viable, because the movement would most likely expand more energy than it could generate from what it collects, but it's at least science based. You could make the flaps solar energy collectors and have it hibernate during times where it's too far away from a sun to collect energy, if you want it's energy consumption to be slightly more realistic.






                  share|improve this answer











                  $endgroup$



                  The creature is actually a giant bussard ramjet. It just looks like it's swimming, because the flaps are used to move particles from the surrounding area towards the body, where they are collected by polyp-like areas covering the body. They are then moved towards an organ that holds the particles until they are required for energy generation or used as propulsion matter. The body could also emit an EM field that moves particles closer to the body, so that the flaps can collect more of it.



                  The end result would be a movement similar to the breaststroke of a swimmer with a forward movement of the flaps close to the body to avoid pushing particles away and a wide backwards movement to collect a large volume of particles.



                  It's probably not scientifically viable, because the movement would most likely expand more energy than it could generate from what it collects, but it's at least science based. You could make the flaps solar energy collectors and have it hibernate during times where it's too far away from a sun to collect energy, if you want it's energy consumption to be slightly more realistic.







                  share|improve this answer














                  share|improve this answer



                  share|improve this answer








                  edited Nov 15 '18 at 7:17

























                  answered Nov 14 '18 at 9:04









                  MorfildurMorfildur

                  63111




                  63111











                  • $begingroup$
                    L.Dutch covered the size of said ramjet in their answer. Not exactly, but the sale math applies I think. IANARS.
                    $endgroup$
                    – Draco18s
                    Nov 14 '18 at 16:51






                  • 1




                    $begingroup$
                    @Draco18s Nah, that's talking about different things. Bussard Ramjets use completely different principles and function at much more reasonable scales (although last I heard people thought they would probably work better as brakes than engines!)
                    $endgroup$
                    – Tim B
                    Nov 14 '18 at 17:22






                  • 1




                    $begingroup$
                    @TimB I haven't seen the science, and IANARS, so I'm just guessing. Point is, space is still ridiculously not-dense (the densest nebulae are less dense than the best laboratory vacuum chamber).
                    $endgroup$
                    – Draco18s
                    Nov 14 '18 at 17:28






                  • 1




                    $begingroup$
                    @Draco18s Wings use fluid dynamics, that's what the other answer covers. Bussard Ramjets use electromagnetic fields to gather interstellar medium to then use as fuel and propellant. It's a completely different mechanism.
                    $endgroup$
                    – Tim B
                    Nov 14 '18 at 17:30
















                  • $begingroup$
                    L.Dutch covered the size of said ramjet in their answer. Not exactly, but the sale math applies I think. IANARS.
                    $endgroup$
                    – Draco18s
                    Nov 14 '18 at 16:51






                  • 1




                    $begingroup$
                    @Draco18s Nah, that's talking about different things. Bussard Ramjets use completely different principles and function at much more reasonable scales (although last I heard people thought they would probably work better as brakes than engines!)
                    $endgroup$
                    – Tim B
                    Nov 14 '18 at 17:22






                  • 1




                    $begingroup$
                    @TimB I haven't seen the science, and IANARS, so I'm just guessing. Point is, space is still ridiculously not-dense (the densest nebulae are less dense than the best laboratory vacuum chamber).
                    $endgroup$
                    – Draco18s
                    Nov 14 '18 at 17:28






                  • 1




                    $begingroup$
                    @Draco18s Wings use fluid dynamics, that's what the other answer covers. Bussard Ramjets use electromagnetic fields to gather interstellar medium to then use as fuel and propellant. It's a completely different mechanism.
                    $endgroup$
                    – Tim B
                    Nov 14 '18 at 17:30















                  $begingroup$
                  L.Dutch covered the size of said ramjet in their answer. Not exactly, but the sale math applies I think. IANARS.
                  $endgroup$
                  – Draco18s
                  Nov 14 '18 at 16:51




                  $begingroup$
                  L.Dutch covered the size of said ramjet in their answer. Not exactly, but the sale math applies I think. IANARS.
                  $endgroup$
                  – Draco18s
                  Nov 14 '18 at 16:51




                  1




                  1




                  $begingroup$
                  @Draco18s Nah, that's talking about different things. Bussard Ramjets use completely different principles and function at much more reasonable scales (although last I heard people thought they would probably work better as brakes than engines!)
                  $endgroup$
                  – Tim B
                  Nov 14 '18 at 17:22




                  $begingroup$
                  @Draco18s Nah, that's talking about different things. Bussard Ramjets use completely different principles and function at much more reasonable scales (although last I heard people thought they would probably work better as brakes than engines!)
                  $endgroup$
                  – Tim B
                  Nov 14 '18 at 17:22




                  1




                  1




                  $begingroup$
                  @TimB I haven't seen the science, and IANARS, so I'm just guessing. Point is, space is still ridiculously not-dense (the densest nebulae are less dense than the best laboratory vacuum chamber).
                  $endgroup$
                  – Draco18s
                  Nov 14 '18 at 17:28




                  $begingroup$
                  @TimB I haven't seen the science, and IANARS, so I'm just guessing. Point is, space is still ridiculously not-dense (the densest nebulae are less dense than the best laboratory vacuum chamber).
                  $endgroup$
                  – Draco18s
                  Nov 14 '18 at 17:28




                  1




                  1




                  $begingroup$
                  @Draco18s Wings use fluid dynamics, that's what the other answer covers. Bussard Ramjets use electromagnetic fields to gather interstellar medium to then use as fuel and propellant. It's a completely different mechanism.
                  $endgroup$
                  – Tim B
                  Nov 14 '18 at 17:30




                  $begingroup$
                  @Draco18s Wings use fluid dynamics, that's what the other answer covers. Bussard Ramjets use electromagnetic fields to gather interstellar medium to then use as fuel and propellant. It's a completely different mechanism.
                  $endgroup$
                  – Tim B
                  Nov 14 '18 at 17:30











                  5












                  $begingroup$

                  Surprisingly a creature swimming in space would be possible, in the sense that it would be able to change its position/location, but not is momentum which would make it useless for long distance travel.



                  The concept is called swimming in spacetime and is the only reactionless propulsion idea we have that would not violate physics.



                  According to wikipedia:




                  “Swimming in spacetime” is a geometrical motive principle that exploits the curved spacetime metric of the gravitational field to permit an extended body undergoing specific deformations in shape, to change position. In weak gravitational fields, like that of Earth, the change in position per deformation cycle would be far too small to detect, but the concept remains of interest as the only unambiguous example of reactionless motion in mainstream physics.
                  https://en.wikipedia.org/wiki/Reactionless_drive#Movement_without_thrust




                  By my understanding, it works through the fact the gravitational field is a gradient (slightly different in different positions, e.g. weaker further away from the source.). By shaping the body so that the center of mass of that body is in a different location where the gravitational field is stronger or weaker and then going back to the original shape, the body would have changed location ever so slightly compared to the previous location. This, however, does not impart any momentum on the body, as the force necessary to take on the shape is exactly the same force as the force to go back to the original shape (That's how I understood the theory, but the more I think about it, the forces required to change shapes should be different in different gravities.).



                  Here's a question on stackexchange on the topic: https://physics.stackexchange.com/questions/886/swimming-in-spacetime-apparent-conserved-quantity-violation
                  Here is a paper on the topic: http://science.sciencemag.org/content/299/5614/1865






                  share|improve this answer









                  $endgroup$

















                    5












                    $begingroup$

                    Surprisingly a creature swimming in space would be possible, in the sense that it would be able to change its position/location, but not is momentum which would make it useless for long distance travel.



                    The concept is called swimming in spacetime and is the only reactionless propulsion idea we have that would not violate physics.



                    According to wikipedia:




                    “Swimming in spacetime” is a geometrical motive principle that exploits the curved spacetime metric of the gravitational field to permit an extended body undergoing specific deformations in shape, to change position. In weak gravitational fields, like that of Earth, the change in position per deformation cycle would be far too small to detect, but the concept remains of interest as the only unambiguous example of reactionless motion in mainstream physics.
                    https://en.wikipedia.org/wiki/Reactionless_drive#Movement_without_thrust




                    By my understanding, it works through the fact the gravitational field is a gradient (slightly different in different positions, e.g. weaker further away from the source.). By shaping the body so that the center of mass of that body is in a different location where the gravitational field is stronger or weaker and then going back to the original shape, the body would have changed location ever so slightly compared to the previous location. This, however, does not impart any momentum on the body, as the force necessary to take on the shape is exactly the same force as the force to go back to the original shape (That's how I understood the theory, but the more I think about it, the forces required to change shapes should be different in different gravities.).



                    Here's a question on stackexchange on the topic: https://physics.stackexchange.com/questions/886/swimming-in-spacetime-apparent-conserved-quantity-violation
                    Here is a paper on the topic: http://science.sciencemag.org/content/299/5614/1865






                    share|improve this answer









                    $endgroup$















                      5












                      5








                      5





                      $begingroup$

                      Surprisingly a creature swimming in space would be possible, in the sense that it would be able to change its position/location, but not is momentum which would make it useless for long distance travel.



                      The concept is called swimming in spacetime and is the only reactionless propulsion idea we have that would not violate physics.



                      According to wikipedia:




                      “Swimming in spacetime” is a geometrical motive principle that exploits the curved spacetime metric of the gravitational field to permit an extended body undergoing specific deformations in shape, to change position. In weak gravitational fields, like that of Earth, the change in position per deformation cycle would be far too small to detect, but the concept remains of interest as the only unambiguous example of reactionless motion in mainstream physics.
                      https://en.wikipedia.org/wiki/Reactionless_drive#Movement_without_thrust




                      By my understanding, it works through the fact the gravitational field is a gradient (slightly different in different positions, e.g. weaker further away from the source.). By shaping the body so that the center of mass of that body is in a different location where the gravitational field is stronger or weaker and then going back to the original shape, the body would have changed location ever so slightly compared to the previous location. This, however, does not impart any momentum on the body, as the force necessary to take on the shape is exactly the same force as the force to go back to the original shape (That's how I understood the theory, but the more I think about it, the forces required to change shapes should be different in different gravities.).



                      Here's a question on stackexchange on the topic: https://physics.stackexchange.com/questions/886/swimming-in-spacetime-apparent-conserved-quantity-violation
                      Here is a paper on the topic: http://science.sciencemag.org/content/299/5614/1865






                      share|improve this answer









                      $endgroup$



                      Surprisingly a creature swimming in space would be possible, in the sense that it would be able to change its position/location, but not is momentum which would make it useless for long distance travel.



                      The concept is called swimming in spacetime and is the only reactionless propulsion idea we have that would not violate physics.



                      According to wikipedia:




                      “Swimming in spacetime” is a geometrical motive principle that exploits the curved spacetime metric of the gravitational field to permit an extended body undergoing specific deformations in shape, to change position. In weak gravitational fields, like that of Earth, the change in position per deformation cycle would be far too small to detect, but the concept remains of interest as the only unambiguous example of reactionless motion in mainstream physics.
                      https://en.wikipedia.org/wiki/Reactionless_drive#Movement_without_thrust




                      By my understanding, it works through the fact the gravitational field is a gradient (slightly different in different positions, e.g. weaker further away from the source.). By shaping the body so that the center of mass of that body is in a different location where the gravitational field is stronger or weaker and then going back to the original shape, the body would have changed location ever so slightly compared to the previous location. This, however, does not impart any momentum on the body, as the force necessary to take on the shape is exactly the same force as the force to go back to the original shape (That's how I understood the theory, but the more I think about it, the forces required to change shapes should be different in different gravities.).



                      Here's a question on stackexchange on the topic: https://physics.stackexchange.com/questions/886/swimming-in-spacetime-apparent-conserved-quantity-violation
                      Here is a paper on the topic: http://science.sciencemag.org/content/299/5614/1865







                      share|improve this answer












                      share|improve this answer



                      share|improve this answer










                      answered Nov 14 '18 at 12:17









                      SyzygySyzygy

                      83128




                      83128





















                          4












                          $begingroup$

                          Perhaps gravity, like that of the space probes. Or they could wear suits of magnets that attract or repel a planets or asteroids magnetic field. If they are some kind of animal perhaps they have a developed gland for an electrical field like an eel. Another idea is that they detect and capture neutrinos from the photons from stars and use them as a connective tissue or string. There are probably many different ideas of how they move because I’m pretty sure they cannot sit still in space. I would probably go with some type of jet stream orbit, and here is where their dust food would accumulate anyway.






                          share|improve this answer









                          $endgroup$












                          • $begingroup$
                            Yes, some good ideas and they probably use all those methods. However I specifically want them to swim in addition to anything else, so propulsion by body movement is what I'm after.
                            $endgroup$
                            – chasly from UK
                            Nov 13 '18 at 18:51















                          4












                          $begingroup$

                          Perhaps gravity, like that of the space probes. Or they could wear suits of magnets that attract or repel a planets or asteroids magnetic field. If they are some kind of animal perhaps they have a developed gland for an electrical field like an eel. Another idea is that they detect and capture neutrinos from the photons from stars and use them as a connective tissue or string. There are probably many different ideas of how they move because I’m pretty sure they cannot sit still in space. I would probably go with some type of jet stream orbit, and here is where their dust food would accumulate anyway.






                          share|improve this answer









                          $endgroup$












                          • $begingroup$
                            Yes, some good ideas and they probably use all those methods. However I specifically want them to swim in addition to anything else, so propulsion by body movement is what I'm after.
                            $endgroup$
                            – chasly from UK
                            Nov 13 '18 at 18:51













                          4












                          4








                          4





                          $begingroup$

                          Perhaps gravity, like that of the space probes. Or they could wear suits of magnets that attract or repel a planets or asteroids magnetic field. If they are some kind of animal perhaps they have a developed gland for an electrical field like an eel. Another idea is that they detect and capture neutrinos from the photons from stars and use them as a connective tissue or string. There are probably many different ideas of how they move because I’m pretty sure they cannot sit still in space. I would probably go with some type of jet stream orbit, and here is where their dust food would accumulate anyway.






                          share|improve this answer









                          $endgroup$



                          Perhaps gravity, like that of the space probes. Or they could wear suits of magnets that attract or repel a planets or asteroids magnetic field. If they are some kind of animal perhaps they have a developed gland for an electrical field like an eel. Another idea is that they detect and capture neutrinos from the photons from stars and use them as a connective tissue or string. There are probably many different ideas of how they move because I’m pretty sure they cannot sit still in space. I would probably go with some type of jet stream orbit, and here is where their dust food would accumulate anyway.







                          share|improve this answer












                          share|improve this answer



                          share|improve this answer










                          answered Nov 13 '18 at 18:31









                          RobusRobus

                          1527




                          1527











                          • $begingroup$
                            Yes, some good ideas and they probably use all those methods. However I specifically want them to swim in addition to anything else, so propulsion by body movement is what I'm after.
                            $endgroup$
                            – chasly from UK
                            Nov 13 '18 at 18:51
















                          • $begingroup$
                            Yes, some good ideas and they probably use all those methods. However I specifically want them to swim in addition to anything else, so propulsion by body movement is what I'm after.
                            $endgroup$
                            – chasly from UK
                            Nov 13 '18 at 18:51















                          $begingroup$
                          Yes, some good ideas and they probably use all those methods. However I specifically want them to swim in addition to anything else, so propulsion by body movement is what I'm after.
                          $endgroup$
                          – chasly from UK
                          Nov 13 '18 at 18:51




                          $begingroup$
                          Yes, some good ideas and they probably use all those methods. However I specifically want them to swim in addition to anything else, so propulsion by body movement is what I'm after.
                          $endgroup$
                          – chasly from UK
                          Nov 13 '18 at 18:51











                          4












                          $begingroup$

                          They are not actually swimming - their extremities are actually EM drives which allow for reactionless motion.






                          share|improve this answer









                          $endgroup$








                          • 2




                            $begingroup$
                            You mean the Impossible Drive, which has never been shown to work in theory or in practice?
                            $endgroup$
                            – Nuclear Wang
                            Nov 13 '18 at 18:51






                          • 2




                            $begingroup$
                            EM Drives have no basis in science. It's like saying that magic exist because it was not proved not to exist.
                            $endgroup$
                            – Antzi
                            Nov 14 '18 at 3:12






                          • 2




                            $begingroup$
                            @Onyz All experiments showing thrust have either been irreproducible, flawed in design, or both.
                            $endgroup$
                            – Nuclear Wang
                            Nov 14 '18 at 14:35






                          • 1




                            $begingroup$
                            @Onyz Scott Manley did an update a few months ago, and still nothing, unfortunately
                            $endgroup$
                            – Aaron F
                            Nov 14 '18 at 15:14






                          • 5




                            $begingroup$
                            @Eth "breaking some of the most fundamental laws of physics" and even "would topple our entire understanding of science overnight" are not reasons to suggest that something does not work as advertised. If it works as advertised, then "physics is wrong" is a better answer than "that thing is not doing what we see it doing". The reason that the drive can be disbelieved is wholly and entirely different: it can be disbelieved because it has not been shown to work as advertised, and that is the primary reason.
                            $endgroup$
                            – Aaron
                            Nov 14 '18 at 17:12















                          4












                          $begingroup$

                          They are not actually swimming - their extremities are actually EM drives which allow for reactionless motion.






                          share|improve this answer









                          $endgroup$








                          • 2




                            $begingroup$
                            You mean the Impossible Drive, which has never been shown to work in theory or in practice?
                            $endgroup$
                            – Nuclear Wang
                            Nov 13 '18 at 18:51






                          • 2




                            $begingroup$
                            EM Drives have no basis in science. It's like saying that magic exist because it was not proved not to exist.
                            $endgroup$
                            – Antzi
                            Nov 14 '18 at 3:12






                          • 2




                            $begingroup$
                            @Onyz All experiments showing thrust have either been irreproducible, flawed in design, or both.
                            $endgroup$
                            – Nuclear Wang
                            Nov 14 '18 at 14:35






                          • 1




                            $begingroup$
                            @Onyz Scott Manley did an update a few months ago, and still nothing, unfortunately
                            $endgroup$
                            – Aaron F
                            Nov 14 '18 at 15:14






                          • 5




                            $begingroup$
                            @Eth "breaking some of the most fundamental laws of physics" and even "would topple our entire understanding of science overnight" are not reasons to suggest that something does not work as advertised. If it works as advertised, then "physics is wrong" is a better answer than "that thing is not doing what we see it doing". The reason that the drive can be disbelieved is wholly and entirely different: it can be disbelieved because it has not been shown to work as advertised, and that is the primary reason.
                            $endgroup$
                            – Aaron
                            Nov 14 '18 at 17:12













                          4












                          4








                          4





                          $begingroup$

                          They are not actually swimming - their extremities are actually EM drives which allow for reactionless motion.






                          share|improve this answer









                          $endgroup$



                          They are not actually swimming - their extremities are actually EM drives which allow for reactionless motion.







                          share|improve this answer












                          share|improve this answer



                          share|improve this answer










                          answered Nov 13 '18 at 18:34









                          AlexanderAlexander

                          19.7k53275




                          19.7k53275







                          • 2




                            $begingroup$
                            You mean the Impossible Drive, which has never been shown to work in theory or in practice?
                            $endgroup$
                            – Nuclear Wang
                            Nov 13 '18 at 18:51






                          • 2




                            $begingroup$
                            EM Drives have no basis in science. It's like saying that magic exist because it was not proved not to exist.
                            $endgroup$
                            – Antzi
                            Nov 14 '18 at 3:12






                          • 2




                            $begingroup$
                            @Onyz All experiments showing thrust have either been irreproducible, flawed in design, or both.
                            $endgroup$
                            – Nuclear Wang
                            Nov 14 '18 at 14:35






                          • 1




                            $begingroup$
                            @Onyz Scott Manley did an update a few months ago, and still nothing, unfortunately
                            $endgroup$
                            – Aaron F
                            Nov 14 '18 at 15:14






                          • 5




                            $begingroup$
                            @Eth "breaking some of the most fundamental laws of physics" and even "would topple our entire understanding of science overnight" are not reasons to suggest that something does not work as advertised. If it works as advertised, then "physics is wrong" is a better answer than "that thing is not doing what we see it doing". The reason that the drive can be disbelieved is wholly and entirely different: it can be disbelieved because it has not been shown to work as advertised, and that is the primary reason.
                            $endgroup$
                            – Aaron
                            Nov 14 '18 at 17:12












                          • 2




                            $begingroup$
                            You mean the Impossible Drive, which has never been shown to work in theory or in practice?
                            $endgroup$
                            – Nuclear Wang
                            Nov 13 '18 at 18:51






                          • 2




                            $begingroup$
                            EM Drives have no basis in science. It's like saying that magic exist because it was not proved not to exist.
                            $endgroup$
                            – Antzi
                            Nov 14 '18 at 3:12






                          • 2




                            $begingroup$
                            @Onyz All experiments showing thrust have either been irreproducible, flawed in design, or both.
                            $endgroup$
                            – Nuclear Wang
                            Nov 14 '18 at 14:35






                          • 1




                            $begingroup$
                            @Onyz Scott Manley did an update a few months ago, and still nothing, unfortunately
                            $endgroup$
                            – Aaron F
                            Nov 14 '18 at 15:14






                          • 5




                            $begingroup$
                            @Eth "breaking some of the most fundamental laws of physics" and even "would topple our entire understanding of science overnight" are not reasons to suggest that something does not work as advertised. If it works as advertised, then "physics is wrong" is a better answer than "that thing is not doing what we see it doing". The reason that the drive can be disbelieved is wholly and entirely different: it can be disbelieved because it has not been shown to work as advertised, and that is the primary reason.
                            $endgroup$
                            – Aaron
                            Nov 14 '18 at 17:12







                          2




                          2




                          $begingroup$
                          You mean the Impossible Drive, which has never been shown to work in theory or in practice?
                          $endgroup$
                          – Nuclear Wang
                          Nov 13 '18 at 18:51




                          $begingroup$
                          You mean the Impossible Drive, which has never been shown to work in theory or in practice?
                          $endgroup$
                          – Nuclear Wang
                          Nov 13 '18 at 18:51




                          2




                          2




                          $begingroup$
                          EM Drives have no basis in science. It's like saying that magic exist because it was not proved not to exist.
                          $endgroup$
                          – Antzi
                          Nov 14 '18 at 3:12




                          $begingroup$
                          EM Drives have no basis in science. It's like saying that magic exist because it was not proved not to exist.
                          $endgroup$
                          – Antzi
                          Nov 14 '18 at 3:12




                          2




                          2




                          $begingroup$
                          @Onyz All experiments showing thrust have either been irreproducible, flawed in design, or both.
                          $endgroup$
                          – Nuclear Wang
                          Nov 14 '18 at 14:35




                          $begingroup$
                          @Onyz All experiments showing thrust have either been irreproducible, flawed in design, or both.
                          $endgroup$
                          – Nuclear Wang
                          Nov 14 '18 at 14:35




                          1




                          1




                          $begingroup$
                          @Onyz Scott Manley did an update a few months ago, and still nothing, unfortunately
                          $endgroup$
                          – Aaron F
                          Nov 14 '18 at 15:14




                          $begingroup$
                          @Onyz Scott Manley did an update a few months ago, and still nothing, unfortunately
                          $endgroup$
                          – Aaron F
                          Nov 14 '18 at 15:14




                          5




                          5




                          $begingroup$
                          @Eth "breaking some of the most fundamental laws of physics" and even "would topple our entire understanding of science overnight" are not reasons to suggest that something does not work as advertised. If it works as advertised, then "physics is wrong" is a better answer than "that thing is not doing what we see it doing". The reason that the drive can be disbelieved is wholly and entirely different: it can be disbelieved because it has not been shown to work as advertised, and that is the primary reason.
                          $endgroup$
                          – Aaron
                          Nov 14 '18 at 17:12




                          $begingroup$
                          @Eth "breaking some of the most fundamental laws of physics" and even "would topple our entire understanding of science overnight" are not reasons to suggest that something does not work as advertised. If it works as advertised, then "physics is wrong" is a better answer than "that thing is not doing what we see it doing". The reason that the drive can be disbelieved is wholly and entirely different: it can be disbelieved because it has not been shown to work as advertised, and that is the primary reason.
                          $endgroup$
                          – Aaron
                          Nov 14 '18 at 17:12











                          4












                          $begingroup$

                          Sorry but you will never be able to swim in there. One m3 of "space" will weight in order 10-21 to 10-25 kg. Your ordinary air is 1.225kg/m3. There is like 1019 grains of sand on Earth. You will destroy your body or run out of food/energy way before you produce some decent speed. Maybe use something like an Ion engine.






                          share|improve this answer











                          $endgroup$

















                            4












                            $begingroup$

                            Sorry but you will never be able to swim in there. One m3 of "space" will weight in order 10-21 to 10-25 kg. Your ordinary air is 1.225kg/m3. There is like 1019 grains of sand on Earth. You will destroy your body or run out of food/energy way before you produce some decent speed. Maybe use something like an Ion engine.






                            share|improve this answer











                            $endgroup$















                              4












                              4








                              4





                              $begingroup$

                              Sorry but you will never be able to swim in there. One m3 of "space" will weight in order 10-21 to 10-25 kg. Your ordinary air is 1.225kg/m3. There is like 1019 grains of sand on Earth. You will destroy your body or run out of food/energy way before you produce some decent speed. Maybe use something like an Ion engine.






                              share|improve this answer











                              $endgroup$



                              Sorry but you will never be able to swim in there. One m3 of "space" will weight in order 10-21 to 10-25 kg. Your ordinary air is 1.225kg/m3. There is like 1019 grains of sand on Earth. You will destroy your body or run out of food/energy way before you produce some decent speed. Maybe use something like an Ion engine.







                              share|improve this answer














                              share|improve this answer



                              share|improve this answer








                              edited Nov 14 '18 at 0:17









                              Renan

                              46.2k11109234




                              46.2k11109234










                              answered Nov 13 '18 at 21:25









                              Artemijs DanilovsArtemijs Danilovs

                              1,704113




                              1,704113





















                                  3












                                  $begingroup$

                                  I can think of two methods of moving through space:




                                  1. Gravity assist—the creatures are highly attuned to gravity wells and know how to position themselves perfectly to slingshot around the universe.


                                  2. Pooping near the speed of light. If you just shoot a tiny poop at 0.999999C, then the energy is ridiculous enough for the "equal and opposite reaction" of Newton's laws to push your creature forward very well indeed.

                                  Actually moving fins against particles in space wont do enough, unless your creature weighs ~1 gram (but that's kinda silly, we'd be moving into fiction from here)



                                  Ok one more bonus point:




                                  1. Super-giant-colossal squid: This is similar to gravity assist, but with more control—tentacles are galaxy-sized in length, and at the end of them is where most of the creature's weight is. By moving the tentacles around, they can shift their weight around closer and farther from various sources of gravity.





                                  share|improve this answer











                                  $endgroup$








                                  • 1




                                    $begingroup$
                                    You mean that the momentum is ridiculous enough, energy is not whats referred to in the third of newtons laws. Though very good answer other than that
                                    $endgroup$
                                    – Ummdustry
                                    Nov 14 '18 at 13:12










                                  • $begingroup$
                                    The tentacles wouldn’t have to be that big. They could merely be lightyears across and move from star to star by ‘reaching out’ in the direction it wants to go, loping lazily from star to star.
                                    $endgroup$
                                    – Joe Bloggs
                                    Nov 14 '18 at 14:01










                                  • $begingroup$
                                    @Ummdustry It's technically correct, though; you need masses of energy to do that.
                                    $endgroup$
                                    – wizzwizz4
                                    Nov 14 '18 at 18:13















                                  3












                                  $begingroup$

                                  I can think of two methods of moving through space:




                                  1. Gravity assist—the creatures are highly attuned to gravity wells and know how to position themselves perfectly to slingshot around the universe.


                                  2. Pooping near the speed of light. If you just shoot a tiny poop at 0.999999C, then the energy is ridiculous enough for the "equal and opposite reaction" of Newton's laws to push your creature forward very well indeed.

                                  Actually moving fins against particles in space wont do enough, unless your creature weighs ~1 gram (but that's kinda silly, we'd be moving into fiction from here)



                                  Ok one more bonus point:




                                  1. Super-giant-colossal squid: This is similar to gravity assist, but with more control—tentacles are galaxy-sized in length, and at the end of them is where most of the creature's weight is. By moving the tentacles around, they can shift their weight around closer and farther from various sources of gravity.





                                  share|improve this answer











                                  $endgroup$








                                  • 1




                                    $begingroup$
                                    You mean that the momentum is ridiculous enough, energy is not whats referred to in the third of newtons laws. Though very good answer other than that
                                    $endgroup$
                                    – Ummdustry
                                    Nov 14 '18 at 13:12










                                  • $begingroup$
                                    The tentacles wouldn’t have to be that big. They could merely be lightyears across and move from star to star by ‘reaching out’ in the direction it wants to go, loping lazily from star to star.
                                    $endgroup$
                                    – Joe Bloggs
                                    Nov 14 '18 at 14:01










                                  • $begingroup$
                                    @Ummdustry It's technically correct, though; you need masses of energy to do that.
                                    $endgroup$
                                    – wizzwizz4
                                    Nov 14 '18 at 18:13













                                  3












                                  3








                                  3





                                  $begingroup$

                                  I can think of two methods of moving through space:




                                  1. Gravity assist—the creatures are highly attuned to gravity wells and know how to position themselves perfectly to slingshot around the universe.


                                  2. Pooping near the speed of light. If you just shoot a tiny poop at 0.999999C, then the energy is ridiculous enough for the "equal and opposite reaction" of Newton's laws to push your creature forward very well indeed.

                                  Actually moving fins against particles in space wont do enough, unless your creature weighs ~1 gram (but that's kinda silly, we'd be moving into fiction from here)



                                  Ok one more bonus point:




                                  1. Super-giant-colossal squid: This is similar to gravity assist, but with more control—tentacles are galaxy-sized in length, and at the end of them is where most of the creature's weight is. By moving the tentacles around, they can shift their weight around closer and farther from various sources of gravity.





                                  share|improve this answer











                                  $endgroup$



                                  I can think of two methods of moving through space:




                                  1. Gravity assist—the creatures are highly attuned to gravity wells and know how to position themselves perfectly to slingshot around the universe.


                                  2. Pooping near the speed of light. If you just shoot a tiny poop at 0.999999C, then the energy is ridiculous enough for the "equal and opposite reaction" of Newton's laws to push your creature forward very well indeed.

                                  Actually moving fins against particles in space wont do enough, unless your creature weighs ~1 gram (but that's kinda silly, we'd be moving into fiction from here)



                                  Ok one more bonus point:




                                  1. Super-giant-colossal squid: This is similar to gravity assist, but with more control—tentacles are galaxy-sized in length, and at the end of them is where most of the creature's weight is. By moving the tentacles around, they can shift their weight around closer and farther from various sources of gravity.






                                  share|improve this answer














                                  share|improve this answer



                                  share|improve this answer








                                  edited Nov 14 '18 at 23:44

























                                  answered Nov 14 '18 at 3:37









                                  Mirror318Mirror318

                                  3,209617




                                  3,209617







                                  • 1




                                    $begingroup$
                                    You mean that the momentum is ridiculous enough, energy is not whats referred to in the third of newtons laws. Though very good answer other than that
                                    $endgroup$
                                    – Ummdustry
                                    Nov 14 '18 at 13:12










                                  • $begingroup$
                                    The tentacles wouldn’t have to be that big. They could merely be lightyears across and move from star to star by ‘reaching out’ in the direction it wants to go, loping lazily from star to star.
                                    $endgroup$
                                    – Joe Bloggs
                                    Nov 14 '18 at 14:01










                                  • $begingroup$
                                    @Ummdustry It's technically correct, though; you need masses of energy to do that.
                                    $endgroup$
                                    – wizzwizz4
                                    Nov 14 '18 at 18:13












                                  • 1




                                    $begingroup$
                                    You mean that the momentum is ridiculous enough, energy is not whats referred to in the third of newtons laws. Though very good answer other than that
                                    $endgroup$
                                    – Ummdustry
                                    Nov 14 '18 at 13:12










                                  • $begingroup$
                                    The tentacles wouldn’t have to be that big. They could merely be lightyears across and move from star to star by ‘reaching out’ in the direction it wants to go, loping lazily from star to star.
                                    $endgroup$
                                    – Joe Bloggs
                                    Nov 14 '18 at 14:01










                                  • $begingroup$
                                    @Ummdustry It's technically correct, though; you need masses of energy to do that.
                                    $endgroup$
                                    – wizzwizz4
                                    Nov 14 '18 at 18:13







                                  1




                                  1




                                  $begingroup$
                                  You mean that the momentum is ridiculous enough, energy is not whats referred to in the third of newtons laws. Though very good answer other than that
                                  $endgroup$
                                  – Ummdustry
                                  Nov 14 '18 at 13:12




                                  $begingroup$
                                  You mean that the momentum is ridiculous enough, energy is not whats referred to in the third of newtons laws. Though very good answer other than that
                                  $endgroup$
                                  – Ummdustry
                                  Nov 14 '18 at 13:12












                                  $begingroup$
                                  The tentacles wouldn’t have to be that big. They could merely be lightyears across and move from star to star by ‘reaching out’ in the direction it wants to go, loping lazily from star to star.
                                  $endgroup$
                                  – Joe Bloggs
                                  Nov 14 '18 at 14:01




                                  $begingroup$
                                  The tentacles wouldn’t have to be that big. They could merely be lightyears across and move from star to star by ‘reaching out’ in the direction it wants to go, loping lazily from star to star.
                                  $endgroup$
                                  – Joe Bloggs
                                  Nov 14 '18 at 14:01












                                  $begingroup$
                                  @Ummdustry It's technically correct, though; you need masses of energy to do that.
                                  $endgroup$
                                  – wizzwizz4
                                  Nov 14 '18 at 18:13




                                  $begingroup$
                                  @Ummdustry It's technically correct, though; you need masses of energy to do that.
                                  $endgroup$
                                  – wizzwizz4
                                  Nov 14 '18 at 18:13











                                  2












                                  $begingroup$

                                  They are not actually swimming.



                                  They are moving extremities in order to manipulate angular momentum. This allows them to change orientation.



                                  See for example https://en.wikipedia.org/wiki/Falling_cat_problem






                                  share|improve this answer









                                  $endgroup$

















                                    2












                                    $begingroup$

                                    They are not actually swimming.



                                    They are moving extremities in order to manipulate angular momentum. This allows them to change orientation.



                                    See for example https://en.wikipedia.org/wiki/Falling_cat_problem






                                    share|improve this answer









                                    $endgroup$















                                      2












                                      2








                                      2





                                      $begingroup$

                                      They are not actually swimming.



                                      They are moving extremities in order to manipulate angular momentum. This allows them to change orientation.



                                      See for example https://en.wikipedia.org/wiki/Falling_cat_problem






                                      share|improve this answer









                                      $endgroup$



                                      They are not actually swimming.



                                      They are moving extremities in order to manipulate angular momentum. This allows them to change orientation.



                                      See for example https://en.wikipedia.org/wiki/Falling_cat_problem







                                      share|improve this answer












                                      share|improve this answer



                                      share|improve this answer










                                      answered Nov 14 '18 at 10:43









                                      TaemyrTaemyr

                                      1,620711




                                      1,620711





















                                          2












                                          $begingroup$

                                          Creature can emit light (or other form of radiation) from its skin cells, e.g. like light bugs or glowing sea creatures.



                                          Emitting light pushes the creature in opposite direction.



                                          Each cell can only emit light for a brief period of time. It takes time to prelenish the chemicals involved, and keeping the light on for too long will cook the cell.



                                          So as the creature sends a wave through its body, it activates lights on the part of the wave that is facing backward. It could probably achieve same effect by spinning, but either it gets dizzy, or it wants to keep its head pointed forward.






                                          share|improve this answer









                                          $endgroup$












                                          • $begingroup$
                                            That's a very novel solution! Although not strictly swimming, the movement would certainly look like it and the light-show would be amazing to watch.
                                            $endgroup$
                                            – chasly from UK
                                            Nov 14 '18 at 17:55
















                                          2












                                          $begingroup$

                                          Creature can emit light (or other form of radiation) from its skin cells, e.g. like light bugs or glowing sea creatures.



                                          Emitting light pushes the creature in opposite direction.



                                          Each cell can only emit light for a brief period of time. It takes time to prelenish the chemicals involved, and keeping the light on for too long will cook the cell.



                                          So as the creature sends a wave through its body, it activates lights on the part of the wave that is facing backward. It could probably achieve same effect by spinning, but either it gets dizzy, or it wants to keep its head pointed forward.






                                          share|improve this answer









                                          $endgroup$












                                          • $begingroup$
                                            That's a very novel solution! Although not strictly swimming, the movement would certainly look like it and the light-show would be amazing to watch.
                                            $endgroup$
                                            – chasly from UK
                                            Nov 14 '18 at 17:55














                                          2












                                          2








                                          2





                                          $begingroup$

                                          Creature can emit light (or other form of radiation) from its skin cells, e.g. like light bugs or glowing sea creatures.



                                          Emitting light pushes the creature in opposite direction.



                                          Each cell can only emit light for a brief period of time. It takes time to prelenish the chemicals involved, and keeping the light on for too long will cook the cell.



                                          So as the creature sends a wave through its body, it activates lights on the part of the wave that is facing backward. It could probably achieve same effect by spinning, but either it gets dizzy, or it wants to keep its head pointed forward.






                                          share|improve this answer









                                          $endgroup$



                                          Creature can emit light (or other form of radiation) from its skin cells, e.g. like light bugs or glowing sea creatures.



                                          Emitting light pushes the creature in opposite direction.



                                          Each cell can only emit light for a brief period of time. It takes time to prelenish the chemicals involved, and keeping the light on for too long will cook the cell.



                                          So as the creature sends a wave through its body, it activates lights on the part of the wave that is facing backward. It could probably achieve same effect by spinning, but either it gets dizzy, or it wants to keep its head pointed forward.







                                          share|improve this answer












                                          share|improve this answer



                                          share|improve this answer










                                          answered Nov 14 '18 at 17:47









                                          Bald BearBald Bear

                                          7,5471127




                                          7,5471127











                                          • $begingroup$
                                            That's a very novel solution! Although not strictly swimming, the movement would certainly look like it and the light-show would be amazing to watch.
                                            $endgroup$
                                            – chasly from UK
                                            Nov 14 '18 at 17:55

















                                          • $begingroup$
                                            That's a very novel solution! Although not strictly swimming, the movement would certainly look like it and the light-show would be amazing to watch.
                                            $endgroup$
                                            – chasly from UK
                                            Nov 14 '18 at 17:55
















                                          $begingroup$
                                          That's a very novel solution! Although not strictly swimming, the movement would certainly look like it and the light-show would be amazing to watch.
                                          $endgroup$
                                          – chasly from UK
                                          Nov 14 '18 at 17:55





                                          $begingroup$
                                          That's a very novel solution! Although not strictly swimming, the movement would certainly look like it and the light-show would be amazing to watch.
                                          $endgroup$
                                          – chasly from UK
                                          Nov 14 '18 at 17:55












                                          1












                                          $begingroup$

                                          Strictly speaking, with the size you have provided and the requirement of "Swimming" (Propulsion through self movement), what you are asking for is impossible. Swimming on our planet only works because we have things of sufficient density to displace to move ourselves forwards. Solar sails don't fit your criteria because they aren't active transportation. There are no active methods that you can use in the vacuum of space to propel yourself with no give/take.



                                          What I would recommend is making this amoeba/whale like a balloon. A thin amoeba shell over its preferred source of food and using the byproducts of digestion to propel slightly. It might take 100000000000 years to reach the next food source at sufficiently low speeds but you would eventually make it to something. maybe.



                                          The things that would work against you at this stage are :



                                          A. Gravity. It would take quite a bit of speed to get out of the solar systems and the eventual pull of the suns.



                                          B. Debris. It may be far and between but there is a good chance of eventually being punctured by debris given the distance and time.






                                          share|improve this answer









                                          $endgroup$












                                          • $begingroup$
                                            100000000000 years to reach the next food source is no problem. They simply go into suspended animation for as long as it takes. They are woken when warmed up by a star or by meeting food particles (interstellar dust).
                                            $endgroup$
                                            – chasly from UK
                                            Nov 14 '18 at 0:26











                                          • $begingroup$
                                            100b is ok? It is like 7 times age of the Universe.
                                            $endgroup$
                                            – Artemijs Danilovs
                                            Nov 14 '18 at 1:18










                                          • $begingroup$
                                            That is some pretty magical hibernation. technically, if the whale heads in the wrong direction it would never reach the next celestial body due to universal expansion.
                                            $endgroup$
                                            – IT Alex
                                            Nov 14 '18 at 12:58















                                          1












                                          $begingroup$

                                          Strictly speaking, with the size you have provided and the requirement of "Swimming" (Propulsion through self movement), what you are asking for is impossible. Swimming on our planet only works because we have things of sufficient density to displace to move ourselves forwards. Solar sails don't fit your criteria because they aren't active transportation. There are no active methods that you can use in the vacuum of space to propel yourself with no give/take.



                                          What I would recommend is making this amoeba/whale like a balloon. A thin amoeba shell over its preferred source of food and using the byproducts of digestion to propel slightly. It might take 100000000000 years to reach the next food source at sufficiently low speeds but you would eventually make it to something. maybe.



                                          The things that would work against you at this stage are :



                                          A. Gravity. It would take quite a bit of speed to get out of the solar systems and the eventual pull of the suns.



                                          B. Debris. It may be far and between but there is a good chance of eventually being punctured by debris given the distance and time.






                                          share|improve this answer









                                          $endgroup$












                                          • $begingroup$
                                            100000000000 years to reach the next food source is no problem. They simply go into suspended animation for as long as it takes. They are woken when warmed up by a star or by meeting food particles (interstellar dust).
                                            $endgroup$
                                            – chasly from UK
                                            Nov 14 '18 at 0:26











                                          • $begingroup$
                                            100b is ok? It is like 7 times age of the Universe.
                                            $endgroup$
                                            – Artemijs Danilovs
                                            Nov 14 '18 at 1:18










                                          • $begingroup$
                                            That is some pretty magical hibernation. technically, if the whale heads in the wrong direction it would never reach the next celestial body due to universal expansion.
                                            $endgroup$
                                            – IT Alex
                                            Nov 14 '18 at 12:58













                                          1












                                          1








                                          1





                                          $begingroup$

                                          Strictly speaking, with the size you have provided and the requirement of "Swimming" (Propulsion through self movement), what you are asking for is impossible. Swimming on our planet only works because we have things of sufficient density to displace to move ourselves forwards. Solar sails don't fit your criteria because they aren't active transportation. There are no active methods that you can use in the vacuum of space to propel yourself with no give/take.



                                          What I would recommend is making this amoeba/whale like a balloon. A thin amoeba shell over its preferred source of food and using the byproducts of digestion to propel slightly. It might take 100000000000 years to reach the next food source at sufficiently low speeds but you would eventually make it to something. maybe.



                                          The things that would work against you at this stage are :



                                          A. Gravity. It would take quite a bit of speed to get out of the solar systems and the eventual pull of the suns.



                                          B. Debris. It may be far and between but there is a good chance of eventually being punctured by debris given the distance and time.






                                          share|improve this answer









                                          $endgroup$



                                          Strictly speaking, with the size you have provided and the requirement of "Swimming" (Propulsion through self movement), what you are asking for is impossible. Swimming on our planet only works because we have things of sufficient density to displace to move ourselves forwards. Solar sails don't fit your criteria because they aren't active transportation. There are no active methods that you can use in the vacuum of space to propel yourself with no give/take.



                                          What I would recommend is making this amoeba/whale like a balloon. A thin amoeba shell over its preferred source of food and using the byproducts of digestion to propel slightly. It might take 100000000000 years to reach the next food source at sufficiently low speeds but you would eventually make it to something. maybe.



                                          The things that would work against you at this stage are :



                                          A. Gravity. It would take quite a bit of speed to get out of the solar systems and the eventual pull of the suns.



                                          B. Debris. It may be far and between but there is a good chance of eventually being punctured by debris given the distance and time.







                                          share|improve this answer












                                          share|improve this answer



                                          share|improve this answer










                                          answered Nov 13 '18 at 20:36









                                          IT AlexIT Alex

                                          7589




                                          7589











                                          • $begingroup$
                                            100000000000 years to reach the next food source is no problem. They simply go into suspended animation for as long as it takes. They are woken when warmed up by a star or by meeting food particles (interstellar dust).
                                            $endgroup$
                                            – chasly from UK
                                            Nov 14 '18 at 0:26











                                          • $begingroup$
                                            100b is ok? It is like 7 times age of the Universe.
                                            $endgroup$
                                            – Artemijs Danilovs
                                            Nov 14 '18 at 1:18










                                          • $begingroup$
                                            That is some pretty magical hibernation. technically, if the whale heads in the wrong direction it would never reach the next celestial body due to universal expansion.
                                            $endgroup$
                                            – IT Alex
                                            Nov 14 '18 at 12:58
















                                          • $begingroup$
                                            100000000000 years to reach the next food source is no problem. They simply go into suspended animation for as long as it takes. They are woken when warmed up by a star or by meeting food particles (interstellar dust).
                                            $endgroup$
                                            – chasly from UK
                                            Nov 14 '18 at 0:26











                                          • $begingroup$
                                            100b is ok? It is like 7 times age of the Universe.
                                            $endgroup$
                                            – Artemijs Danilovs
                                            Nov 14 '18 at 1:18










                                          • $begingroup$
                                            That is some pretty magical hibernation. technically, if the whale heads in the wrong direction it would never reach the next celestial body due to universal expansion.
                                            $endgroup$
                                            – IT Alex
                                            Nov 14 '18 at 12:58















                                          $begingroup$
                                          100000000000 years to reach the next food source is no problem. They simply go into suspended animation for as long as it takes. They are woken when warmed up by a star or by meeting food particles (interstellar dust).
                                          $endgroup$
                                          – chasly from UK
                                          Nov 14 '18 at 0:26





                                          $begingroup$
                                          100000000000 years to reach the next food source is no problem. They simply go into suspended animation for as long as it takes. They are woken when warmed up by a star or by meeting food particles (interstellar dust).
                                          $endgroup$
                                          – chasly from UK
                                          Nov 14 '18 at 0:26













                                          $begingroup$
                                          100b is ok? It is like 7 times age of the Universe.
                                          $endgroup$
                                          – Artemijs Danilovs
                                          Nov 14 '18 at 1:18




                                          $begingroup$
                                          100b is ok? It is like 7 times age of the Universe.
                                          $endgroup$
                                          – Artemijs Danilovs
                                          Nov 14 '18 at 1:18












                                          $begingroup$
                                          That is some pretty magical hibernation. technically, if the whale heads in the wrong direction it would never reach the next celestial body due to universal expansion.
                                          $endgroup$
                                          – IT Alex
                                          Nov 14 '18 at 12:58




                                          $begingroup$
                                          That is some pretty magical hibernation. technically, if the whale heads in the wrong direction it would never reach the next celestial body due to universal expansion.
                                          $endgroup$
                                          – IT Alex
                                          Nov 14 '18 at 12:58











                                          1












                                          $begingroup$

                                          So what discoveries in physics since Newton will enable them to move this way?



                                          NONE



                                          But don't worry, I got you covered.



                                          Your space whale has a huge mouth and baleens that can collect the interstellar particules. Theses are then stocked as food and propellant.



                                          Your whale has a special organ that can expel any matter at very high speeds (read a ion engine). This organ needs electricity to function.



                                          That's where the huge flippers come into play. The space whale start moving its gigantic flippers and using a dedicated organ, stop them almost instantly when they reach top speed.



                                          The moment of inertia energy is converted into electricity used by the organ that actually propel your whale.



                                          Hence, the whale move by pulse and appear to swim into space, leaving a trail of space dust behind it. It goes into hibernation for long periods of time, in order to get enough propellant to accumulate in its mouth.






                                          share|improve this answer









                                          $endgroup$

















                                            1












                                            $begingroup$

                                            So what discoveries in physics since Newton will enable them to move this way?



                                            NONE



                                            But don't worry, I got you covered.



                                            Your space whale has a huge mouth and baleens that can collect the interstellar particules. Theses are then stocked as food and propellant.



                                            Your whale has a special organ that can expel any matter at very high speeds (read a ion engine). This organ needs electricity to function.



                                            That's where the huge flippers come into play. The space whale start moving its gigantic flippers and using a dedicated organ, stop them almost instantly when they reach top speed.



                                            The moment of inertia energy is converted into electricity used by the organ that actually propel your whale.



                                            Hence, the whale move by pulse and appear to swim into space, leaving a trail of space dust behind it. It goes into hibernation for long periods of time, in order to get enough propellant to accumulate in its mouth.






                                            share|improve this answer









                                            $endgroup$















                                              1












                                              1








                                              1





                                              $begingroup$

                                              So what discoveries in physics since Newton will enable them to move this way?



                                              NONE



                                              But don't worry, I got you covered.



                                              Your space whale has a huge mouth and baleens that can collect the interstellar particules. Theses are then stocked as food and propellant.



                                              Your whale has a special organ that can expel any matter at very high speeds (read a ion engine). This organ needs electricity to function.



                                              That's where the huge flippers come into play. The space whale start moving its gigantic flippers and using a dedicated organ, stop them almost instantly when they reach top speed.



                                              The moment of inertia energy is converted into electricity used by the organ that actually propel your whale.



                                              Hence, the whale move by pulse and appear to swim into space, leaving a trail of space dust behind it. It goes into hibernation for long periods of time, in order to get enough propellant to accumulate in its mouth.






                                              share|improve this answer









                                              $endgroup$



                                              So what discoveries in physics since Newton will enable them to move this way?



                                              NONE



                                              But don't worry, I got you covered.



                                              Your space whale has a huge mouth and baleens that can collect the interstellar particules. Theses are then stocked as food and propellant.



                                              Your whale has a special organ that can expel any matter at very high speeds (read a ion engine). This organ needs electricity to function.



                                              That's where the huge flippers come into play. The space whale start moving its gigantic flippers and using a dedicated organ, stop them almost instantly when they reach top speed.



                                              The moment of inertia energy is converted into electricity used by the organ that actually propel your whale.



                                              Hence, the whale move by pulse and appear to swim into space, leaving a trail of space dust behind it. It goes into hibernation for long periods of time, in order to get enough propellant to accumulate in its mouth.







                                              share|improve this answer












                                              share|improve this answer



                                              share|improve this answer










                                              answered Nov 14 '18 at 0:54









                                              AntziAntzi

                                              20316




                                              20316





















                                                  1












                                                  $begingroup$

                                                  They are swimming in space. It's just that their local space is not as much of a near vacuum as our solar system or most of the interstellar medium. The local space happens to be the denser sections of a large, heavy accretion disk that has yet to fuse into a star or form planets. Or they swim when they pass through a binary star system, one star eating the other, with matter streaming from one point to the other giving them enough high speed, low density material to move against.



                                                  If you're attached to them being interstellar, perhaps they return to the "stream" to mate, and the dancing is a ritual toward that goal, forming eddies and whorls that they can sense and find each other. Add "seeking love" and you satisfy your thematic requirements that they dance, and that the act can been seen as beautiful.






                                                  share|improve this answer









                                                  $endgroup$

















                                                    1












                                                    $begingroup$

                                                    They are swimming in space. It's just that their local space is not as much of a near vacuum as our solar system or most of the interstellar medium. The local space happens to be the denser sections of a large, heavy accretion disk that has yet to fuse into a star or form planets. Or they swim when they pass through a binary star system, one star eating the other, with matter streaming from one point to the other giving them enough high speed, low density material to move against.



                                                    If you're attached to them being interstellar, perhaps they return to the "stream" to mate, and the dancing is a ritual toward that goal, forming eddies and whorls that they can sense and find each other. Add "seeking love" and you satisfy your thematic requirements that they dance, and that the act can been seen as beautiful.






                                                    share|improve this answer









                                                    $endgroup$















                                                      1












                                                      1








                                                      1





                                                      $begingroup$

                                                      They are swimming in space. It's just that their local space is not as much of a near vacuum as our solar system or most of the interstellar medium. The local space happens to be the denser sections of a large, heavy accretion disk that has yet to fuse into a star or form planets. Or they swim when they pass through a binary star system, one star eating the other, with matter streaming from one point to the other giving them enough high speed, low density material to move against.



                                                      If you're attached to them being interstellar, perhaps they return to the "stream" to mate, and the dancing is a ritual toward that goal, forming eddies and whorls that they can sense and find each other. Add "seeking love" and you satisfy your thematic requirements that they dance, and that the act can been seen as beautiful.






                                                      share|improve this answer









                                                      $endgroup$



                                                      They are swimming in space. It's just that their local space is not as much of a near vacuum as our solar system or most of the interstellar medium. The local space happens to be the denser sections of a large, heavy accretion disk that has yet to fuse into a star or form planets. Or they swim when they pass through a binary star system, one star eating the other, with matter streaming from one point to the other giving them enough high speed, low density material to move against.



                                                      If you're attached to them being interstellar, perhaps they return to the "stream" to mate, and the dancing is a ritual toward that goal, forming eddies and whorls that they can sense and find each other. Add "seeking love" and you satisfy your thematic requirements that they dance, and that the act can been seen as beautiful.







                                                      share|improve this answer












                                                      share|improve this answer



                                                      share|improve this answer










                                                      answered Nov 14 '18 at 11:36









                                                      Evan EdwardsEvan Edwards

                                                      112




                                                      112





















                                                          1












                                                          $begingroup$

                                                          The 'swimming' motions are actually just used to pump whatever reaction-mass through the creature's body, expelling out the 'rear'.






                                                          share|improve this answer









                                                          $endgroup$

















                                                            1












                                                            $begingroup$

                                                            The 'swimming' motions are actually just used to pump whatever reaction-mass through the creature's body, expelling out the 'rear'.






                                                            share|improve this answer









                                                            $endgroup$















                                                              1












                                                              1








                                                              1





                                                              $begingroup$

                                                              The 'swimming' motions are actually just used to pump whatever reaction-mass through the creature's body, expelling out the 'rear'.






                                                              share|improve this answer









                                                              $endgroup$



                                                              The 'swimming' motions are actually just used to pump whatever reaction-mass through the creature's body, expelling out the 'rear'.







                                                              share|improve this answer












                                                              share|improve this answer



                                                              share|improve this answer










                                                              answered Nov 14 '18 at 16:57









                                                              Theo BrinkmanTheo Brinkman

                                                              1493




                                                              1493





















                                                                  1












                                                                  $begingroup$

                                                                  It is obvious that they will not be the size of a solar system.



                                                                  This means they cannot have fins large enough to be useful in the vacuum of space and cannot swim that way.



                                                                  But if they had a naturally occurring powerful em field projected from their flippers perhaps they could use that em field to act as an invisible fin. Using it to push the atoms in space Around to propel this leviathan through space.



                                                                  How they are able to generate an em field large enough to effect a sufficient mass to use as propellant through space I leave to you.



                                                                  But it would work.






                                                                  share|improve this answer











                                                                  $endgroup$

















                                                                    1












                                                                    $begingroup$

                                                                    It is obvious that they will not be the size of a solar system.



                                                                    This means they cannot have fins large enough to be useful in the vacuum of space and cannot swim that way.



                                                                    But if they had a naturally occurring powerful em field projected from their flippers perhaps they could use that em field to act as an invisible fin. Using it to push the atoms in space Around to propel this leviathan through space.



                                                                    How they are able to generate an em field large enough to effect a sufficient mass to use as propellant through space I leave to you.



                                                                    But it would work.






                                                                    share|improve this answer











                                                                    $endgroup$















                                                                      1












                                                                      1








                                                                      1





                                                                      $begingroup$

                                                                      It is obvious that they will not be the size of a solar system.



                                                                      This means they cannot have fins large enough to be useful in the vacuum of space and cannot swim that way.



                                                                      But if they had a naturally occurring powerful em field projected from their flippers perhaps they could use that em field to act as an invisible fin. Using it to push the atoms in space Around to propel this leviathan through space.



                                                                      How they are able to generate an em field large enough to effect a sufficient mass to use as propellant through space I leave to you.



                                                                      But it would work.






                                                                      share|improve this answer











                                                                      $endgroup$



                                                                      It is obvious that they will not be the size of a solar system.



                                                                      This means they cannot have fins large enough to be useful in the vacuum of space and cannot swim that way.



                                                                      But if they had a naturally occurring powerful em field projected from their flippers perhaps they could use that em field to act as an invisible fin. Using it to push the atoms in space Around to propel this leviathan through space.



                                                                      How they are able to generate an em field large enough to effect a sufficient mass to use as propellant through space I leave to you.



                                                                      But it would work.







                                                                      share|improve this answer














                                                                      share|improve this answer



                                                                      share|improve this answer








                                                                      edited Nov 14 '18 at 17:56









                                                                      Aaron

                                                                      2,494520




                                                                      2,494520










                                                                      answered Nov 13 '18 at 23:56









                                                                      Garret GangGarret Gang

                                                                      38218




                                                                      38218





















                                                                          0












                                                                          $begingroup$

                                                                          How about ion thrusters?



                                                                          The creatures inhale and store relatively small amount of xenon and due to internal biology(the wings are solar collectors?) produce enough electricity to to shoot xenon ions through some orifice in their body to produce thrust. I don't have the number on hand, but I understand that xenon ion thrusters are extremely efficient but they have very low acceleration



                                                                          While this isn't "swimming", the combination of wings (for solar) and thrust might look enough like swimming/flying to the casual observer






                                                                          share|improve this answer











                                                                          $endgroup$












                                                                          • $begingroup$
                                                                            This does not address the OP's request for a mechanical way to "swim" in space.
                                                                            $endgroup$
                                                                            – anon
                                                                            Nov 14 '18 at 14:59










                                                                          • $begingroup$
                                                                            @anon is this better?
                                                                            $endgroup$
                                                                            – Nullman
                                                                            Nov 14 '18 at 15:02















                                                                          0












                                                                          $begingroup$

                                                                          How about ion thrusters?



                                                                          The creatures inhale and store relatively small amount of xenon and due to internal biology(the wings are solar collectors?) produce enough electricity to to shoot xenon ions through some orifice in their body to produce thrust. I don't have the number on hand, but I understand that xenon ion thrusters are extremely efficient but they have very low acceleration



                                                                          While this isn't "swimming", the combination of wings (for solar) and thrust might look enough like swimming/flying to the casual observer






                                                                          share|improve this answer











                                                                          $endgroup$












                                                                          • $begingroup$
                                                                            This does not address the OP's request for a mechanical way to "swim" in space.
                                                                            $endgroup$
                                                                            – anon
                                                                            Nov 14 '18 at 14:59










                                                                          • $begingroup$
                                                                            @anon is this better?
                                                                            $endgroup$
                                                                            – Nullman
                                                                            Nov 14 '18 at 15:02













                                                                          0












                                                                          0








                                                                          0





                                                                          $begingroup$

                                                                          How about ion thrusters?



                                                                          The creatures inhale and store relatively small amount of xenon and due to internal biology(the wings are solar collectors?) produce enough electricity to to shoot xenon ions through some orifice in their body to produce thrust. I don't have the number on hand, but I understand that xenon ion thrusters are extremely efficient but they have very low acceleration



                                                                          While this isn't "swimming", the combination of wings (for solar) and thrust might look enough like swimming/flying to the casual observer






                                                                          share|improve this answer











                                                                          $endgroup$



                                                                          How about ion thrusters?



                                                                          The creatures inhale and store relatively small amount of xenon and due to internal biology(the wings are solar collectors?) produce enough electricity to to shoot xenon ions through some orifice in their body to produce thrust. I don't have the number on hand, but I understand that xenon ion thrusters are extremely efficient but they have very low acceleration



                                                                          While this isn't "swimming", the combination of wings (for solar) and thrust might look enough like swimming/flying to the casual observer







                                                                          share|improve this answer














                                                                          share|improve this answer



                                                                          share|improve this answer








                                                                          edited Nov 14 '18 at 15:02

























                                                                          answered Nov 14 '18 at 14:54









                                                                          NullmanNullman

                                                                          1012




                                                                          1012











                                                                          • $begingroup$
                                                                            This does not address the OP's request for a mechanical way to "swim" in space.
                                                                            $endgroup$
                                                                            – anon
                                                                            Nov 14 '18 at 14:59










                                                                          • $begingroup$
                                                                            @anon is this better?
                                                                            $endgroup$
                                                                            – Nullman
                                                                            Nov 14 '18 at 15:02
















                                                                          • $begingroup$
                                                                            This does not address the OP's request for a mechanical way to "swim" in space.
                                                                            $endgroup$
                                                                            – anon
                                                                            Nov 14 '18 at 14:59










                                                                          • $begingroup$
                                                                            @anon is this better?
                                                                            $endgroup$
                                                                            – Nullman
                                                                            Nov 14 '18 at 15:02















                                                                          $begingroup$
                                                                          This does not address the OP's request for a mechanical way to "swim" in space.
                                                                          $endgroup$
                                                                          – anon
                                                                          Nov 14 '18 at 14:59




                                                                          $begingroup$
                                                                          This does not address the OP's request for a mechanical way to "swim" in space.
                                                                          $endgroup$
                                                                          – anon
                                                                          Nov 14 '18 at 14:59












                                                                          $begingroup$
                                                                          @anon is this better?
                                                                          $endgroup$
                                                                          – Nullman
                                                                          Nov 14 '18 at 15:02




                                                                          $begingroup$
                                                                          @anon is this better?
                                                                          $endgroup$
                                                                          – Nullman
                                                                          Nov 14 '18 at 15:02











                                                                          0












                                                                          $begingroup$

                                                                          If you want to get more speculative, they swim through the "Luminiferous aether".



                                                                          The aether was a theoretical medium through which light was thought to propagate, thus explaining its wave-like nature. The theory was discarded as ever-more-sensitive experiments failed to detect any ripples in the apparent speed of light caused by the momentum of Earth not matching the momentum of the aether...



                                                                          Of course, we have since discovered that all of the elementary particles of matter also travel as waves. (This is how electron microscopes work) The wavelength merely decreases as the particles get larger until it is too short for us to detect. Which would offer a theoretical alternative explanation for why the experiments detected no variation between light and matter due to aetheric ripples: The test equipment itself is also an aetheric ripple and therefore there can never be a difference in medium velocity between light and matter.



                                                                          Your creatures have found some way to push against the aether itself. (Most likely the creatures are actually the ripples caused by the actions of a higher-level entity that is not entirely perceptible to our senses.) This would allow them to "swim" through seemingly empty (ripple-free) space.



                                                                          There are a few potential side-effects to this:



                                                                          Such creatures could be a source of extra-universal energy to keep the universe running (similar to the theorized, but never discovered "white hole").



                                                                          Such creatures would be theoretically capable of exceeding the speed of light.



                                                                          Such creatures would likely be harmed only by (from our point of view) exceedingly violent events. (If all matter is ripples in the aether, they are the water-bug swimming on the surface. It would take some really big ripples to drown them.)



                                                                          Such creatures might appear spontaneously in relatively calm places and disappear in turbulent places. (The swimmers get out of the pool when the water gets too churned up.)






                                                                          share|improve this answer









                                                                          $endgroup$

















                                                                            0












                                                                            $begingroup$

                                                                            If you want to get more speculative, they swim through the "Luminiferous aether".



                                                                            The aether was a theoretical medium through which light was thought to propagate, thus explaining its wave-like nature. The theory was discarded as ever-more-sensitive experiments failed to detect any ripples in the apparent speed of light caused by the momentum of Earth not matching the momentum of the aether...



                                                                            Of course, we have since discovered that all of the elementary particles of matter also travel as waves. (This is how electron microscopes work) The wavelength merely decreases as the particles get larger until it is too short for us to detect. Which would offer a theoretical alternative explanation for why the experiments detected no variation between light and matter due to aetheric ripples: The test equipment itself is also an aetheric ripple and therefore there can never be a difference in medium velocity between light and matter.



                                                                            Your creatures have found some way to push against the aether itself. (Most likely the creatures are actually the ripples caused by the actions of a higher-level entity that is not entirely perceptible to our senses.) This would allow them to "swim" through seemingly empty (ripple-free) space.



                                                                            There are a few potential side-effects to this:



                                                                            Such creatures could be a source of extra-universal energy to keep the universe running (similar to the theorized, but never discovered "white hole").



                                                                            Such creatures would be theoretically capable of exceeding the speed of light.



                                                                            Such creatures would likely be harmed only by (from our point of view) exceedingly violent events. (If all matter is ripples in the aether, they are the water-bug swimming on the surface. It would take some really big ripples to drown them.)



                                                                            Such creatures might appear spontaneously in relatively calm places and disappear in turbulent places. (The swimmers get out of the pool when the water gets too churned up.)






                                                                            share|improve this answer









                                                                            $endgroup$















                                                                              0












                                                                              0








                                                                              0





                                                                              $begingroup$

                                                                              If you want to get more speculative, they swim through the "Luminiferous aether".



                                                                              The aether was a theoretical medium through which light was thought to propagate, thus explaining its wave-like nature. The theory was discarded as ever-more-sensitive experiments failed to detect any ripples in the apparent speed of light caused by the momentum of Earth not matching the momentum of the aether...



                                                                              Of course, we have since discovered that all of the elementary particles of matter also travel as waves. (This is how electron microscopes work) The wavelength merely decreases as the particles get larger until it is too short for us to detect. Which would offer a theoretical alternative explanation for why the experiments detected no variation between light and matter due to aetheric ripples: The test equipment itself is also an aetheric ripple and therefore there can never be a difference in medium velocity between light and matter.



                                                                              Your creatures have found some way to push against the aether itself. (Most likely the creatures are actually the ripples caused by the actions of a higher-level entity that is not entirely perceptible to our senses.) This would allow them to "swim" through seemingly empty (ripple-free) space.



                                                                              There are a few potential side-effects to this:



                                                                              Such creatures could be a source of extra-universal energy to keep the universe running (similar to the theorized, but never discovered "white hole").



                                                                              Such creatures would be theoretically capable of exceeding the speed of light.



                                                                              Such creatures would likely be harmed only by (from our point of view) exceedingly violent events. (If all matter is ripples in the aether, they are the water-bug swimming on the surface. It would take some really big ripples to drown them.)



                                                                              Such creatures might appear spontaneously in relatively calm places and disappear in turbulent places. (The swimmers get out of the pool when the water gets too churned up.)






                                                                              share|improve this answer









                                                                              $endgroup$



                                                                              If you want to get more speculative, they swim through the "Luminiferous aether".



                                                                              The aether was a theoretical medium through which light was thought to propagate, thus explaining its wave-like nature. The theory was discarded as ever-more-sensitive experiments failed to detect any ripples in the apparent speed of light caused by the momentum of Earth not matching the momentum of the aether...



                                                                              Of course, we have since discovered that all of the elementary particles of matter also travel as waves. (This is how electron microscopes work) The wavelength merely decreases as the particles get larger until it is too short for us to detect. Which would offer a theoretical alternative explanation for why the experiments detected no variation between light and matter due to aetheric ripples: The test equipment itself is also an aetheric ripple and therefore there can never be a difference in medium velocity between light and matter.



                                                                              Your creatures have found some way to push against the aether itself. (Most likely the creatures are actually the ripples caused by the actions of a higher-level entity that is not entirely perceptible to our senses.) This would allow them to "swim" through seemingly empty (ripple-free) space.



                                                                              There are a few potential side-effects to this:



                                                                              Such creatures could be a source of extra-universal energy to keep the universe running (similar to the theorized, but never discovered "white hole").



                                                                              Such creatures would be theoretically capable of exceeding the speed of light.



                                                                              Such creatures would likely be harmed only by (from our point of view) exceedingly violent events. (If all matter is ripples in the aether, they are the water-bug swimming on the surface. It would take some really big ripples to drown them.)



                                                                              Such creatures might appear spontaneously in relatively calm places and disappear in turbulent places. (The swimmers get out of the pool when the water gets too churned up.)







                                                                              share|improve this answer












                                                                              share|improve this answer



                                                                              share|improve this answer










                                                                              answered Nov 14 '18 at 20:27









                                                                              PerkinsPerkins

                                                                              3,352515




                                                                              3,352515



























                                                                                  draft saved

                                                                                  draft discarded
















































                                                                                  Thanks for contributing an answer to Worldbuilding Stack Exchange!


                                                                                  • Please be sure to answer the question. Provide details and share your research!

                                                                                  But avoid


                                                                                  • Asking for help, clarification, or responding to other answers.

                                                                                  • Making statements based on opinion; back them up with references or personal experience.

                                                                                  Use MathJax to format equations. MathJax reference.


                                                                                  To learn more, see our tips on writing great answers.




                                                                                  draft saved


                                                                                  draft discarded














                                                                                  StackExchange.ready(
                                                                                  function ()
                                                                                  StackExchange.openid.initPostLogin('.new-post-login', 'https%3a%2f%2fworldbuilding.stackexchange.com%2fquestions%2f130160%2fswimming-in-space-what-is-the-mechanism%23new-answer', 'question_page');

                                                                                  );

                                                                                  Post as a guest















                                                                                  Required, but never shown





















































                                                                                  Required, but never shown














                                                                                  Required, but never shown












                                                                                  Required, but never shown







                                                                                  Required, but never shown

































                                                                                  Required, but never shown














                                                                                  Required, but never shown












                                                                                  Required, but never shown







                                                                                  Required, but never shown







                                                                                  Popular posts from this blog

                                                                                  Top Tejano songwriter Luis Silva dead of heart attack at 64

                                                                                  ReactJS Fetched API data displays live - need Data displayed static

                                                                                  政党